Archivo de la etiqueta: Cirugía Ortopédica y Traumatología

Preguntas MIR enero 2024: Cirugía Ortopédica y Traumatología.

Respuestas cortesía de Cristina Ojeda Thies (@ojedathies), Antonio Sánchez Fernández (@AnSanFer_Dr) y Miguel Vázquez Gómez (@miguelvazquezdr

Estas respuestas no son las oficiales ( las actualizaremos con las mismas en cuanto salgan) sólo reflejan la opinión de aquellos voluntarios que han elegido participar en la Iniciativa MIR 2.0 de respuesta al examen MIR de enero de 2024. (La numeración se corresponde con la Versión 0 del examen)

Ahora te toca a ti participar para que esta iniciativa siga perdurando. Ya sea ayudando a responder una parcela de preguntas, o simplemente difundiendo la iniciativa entre tus compañeros y conocidos. Colabora pulsando aquí.

Índice de contenidos

21. Pregunta asociada a la imagen 21.

Un paciente de 27 años, tras sufrir una caída de un caballo, presenta un traumatismo en el miembro inferior izquierdo con una actitud del mismo en flexión, aducción y rotación interna. Tras realizarle la radiografía anteroposterior de pelvis que se muestra, señale cuál es el diagnóstico:

1. Fractura pertrocantérea de fémur izquierdo.

2. Fractura subcapital de fémur izquierdo.

3. Luxación anterior de cadera, con fractura de pared anterior de acetábulo.

4. Luxación posterior de cadera, con fractura de pared posterior de acetábulo.

Respuesta 4. 

Pregunta que no debería dar mucho problema. Es una luxación posterior de cadera, con actitud en “bañista  sorprendido”, y una radiografía bastante típica. 

De Martínez, F. M., & Martínez-Aedo, A. L. U. (2022). Traumatología y Ortopedia para el grado en medicina. Elsevier Health Sciences: “Se podrán añadir otras lesiones, tales como la rotura de la ceja posterior en las luxaciones posteriores… “En la luxación posterosuperior o ilíaca, la más frecuente, el miembro se presenta acortado, en flexión de 20°, aducción y rotación interna, posición conocida como de la «bañista sorprendida»”

22. Pregunta asociada a la imagen 22.

Mujer de 75 años, autónoma y activa, que sufre una caída en su domicilio y acude a urgencias con dolor, impotencia funcional y deformidad alrededor de la rodilla izquierda. Había sido intervenida quirúrgicamente de esta rodilla hacía 2 años por gonartrosis mediante una artroplastia total de rodilla cementada postero-estabilizada. Los pulsos distales están conservados. Existe una movilidad patológica de la rodilla izquierda sin apreciarse déficit neurológico en la pierna afecta. Se realizan la radiografía y la TC que se muestran en las imágenes. ¿Cuál de las siguientes sería la actitud terapéutica en esta situación?:

1. Placa condílea de ángulo fijo a 95 grados con tornillos de compresión interfragmentaria.

2. Placa condílea de fémur bloqueada de fijación angular.

3. Clavo de fémur retrógrado acerrojado largo.

4. Clavo anterógrado cérvico-cefálico de fémur para proteger todo el fémur.

Cristina Ojeda: Respuesta 2 

Se trata de una fractura periprotéscia de fémur distal, interesa la mayor estabilización posible de la extremidad con menor agresión quirúrgica, que permita una movilización precoz de la paciente, con el fin de evitar en la medida de lo posible la pérdida de autonomía funcional. Esto se hace ahora con placas anatómicas bloqueadas de fijación angular. 

De Martínez, F. M., & Martínez-Aedo, A. L. U. (2022). Traumatología y Ortopedia para el grado en medicina. Elsevier Health Sciences: “En la actualidad se preconizan nuevos sistemas con placas adaptadas a la morfología zonal (premoldeadas) y tornillos bloqueados a la placa (AxSOS™, Less Invasive Stabilization System [LISS]). Estos sistemas son placas de stabilidad angular y el instrumental para su implanta­ción permite la cirugía mínimamente invasiva o abordajes menos agresivos para la osteosíntesis.”

Antonio Sánchez: Me parece una vergüenza de pregunta. En esta fractura entre especialistas discutiríamos entre una placa, clavo retrógrado o megaprótesis. Y se precisa un estudio minucioso del TAC para tomar la decisión. 

pregunta 29.

De las siguientes afirmaciones relacionadas con las estructuras anatómicas que forman el manguito de los rotadores, señale la correcta:

1. Los músculos que forman el manguito de los rotadores son el subescapular, el supraespinoso, el infraespinoso y el redondo mayor.

2. Los tendones de los músculos del manguito refuerzan la cápsula fibrosa de la articulación

escápulo-humeral.

3. El músculo infraespinoso rota el brazo en dirección interna.

4. El músculo supraespinoso ayuda a la aducción (aproximación) del brazo.

Respuesta 2. 

La opción 1 es falsa, porque no es el redondo mayor sino el MENOR. La 3 también, porque el infraespinoso es un rotador EXTERNO. La 4 también, el supraespinoso es ABDUCTOR. Finalmente, la 2 es cierta. 

De Martínez, F. M., & Martínez-Aedo, A. L. U. (2022). Traumatología y Ortopedia para el grado en medicina. Elsevier Health Sciences: “El manguito se adhiere firmemente a la cápsula glenohumeral subyacente y la refuerza circunferencial­mente, excepto en el intervalo rotador y el receso axilar.” podría valorarse la impugnación por lo del intervalo rotador y el receso axilar, pero las otras son más falsas

pregunta 98.

Paciente de 27 años que sufre un accidente de alto impacto con patinete eléctrico en la calle. A la llegada a urgencias está consciente y orientado, Glasgow 15, con tendencia a la hipotensión. En la exploración inicial presenta impotencia funcional con un balance muscular según Daniels de 5 en todos los grupos de extremidades superiores. En extremidades inferiores psoas 5, cuadriceps 3, tibial anterior, triceps sural y extensores del dedo gordo 0 bilateral. De las siguientes, la sospecha diagnóstica inicial es:

1. Fractura-luxación de C7-T1

2. Fractura estallido de L2 con afectación neurológica.

3. Isquemia medular lumbar secundaria al traumatismo.

4. Fractura estallido de T12 con afectación medular completa.

Respuesta 2 

Se trata de un paciente con una lesión neurológica postraumática, y se valoran los grupos musculares según la escala de Daniels, que va de 0 (ausencia de función) a 5 (función normal). Sólo hay afectación de algunos grupos musculares de los miembros inferiores (cuádriceps parcial, y completa de tibial anterior, triceps sural, extensores del hallux bilateral). 

Las lesiones COMPLETAS tienen una ausencia de función sensitivomotora distal al segmento afecto, pero como el cuádriceps tiene afectación parcial, descartamos la opción 4. También queda descartada la 1 al tener una función completa del psoas. 

El cono medular termina en la porción inferior de L1, y distal a ello están las raíces lumbares y sacras, por lo que una fractura-estallido de L2 con invasión del canal medular puede comprometer las raíces distales a este nivel. 

Aunque la isquemia medular puede dar un síndrome medular anterior con pérdida de función motora debajo del nivel lesionado, es raro que se genere una isquemia medular por trauma, siendo la causa más frecuente la iatrogénica (cirugía aórtica). 

De Martínez, F. M., & Martínez-Aedo, A. L. U. (2022). Traumatología y Ortopedia para el grado en medicina. Elsevier Health Sciences:

“(T12), L1, L2, L3 Flexores de la cadera

L2, L3, L4 Cuádriceps

L4 Tibial anterior

L5 EHL, EDL

S1 Flexores del tobillo, peroneo”

pregunta 99.

Niña de 12 años sin antecedentes relevantes, con menarquia hace 4 meses, que acude a la consulta de rehabilitación derivada por su pediatra tras encontrar una asimetría de hombros en la revisión anual. En la exploración física presenta una asimetría de la altura de los hombros y de las caderas con test de Adams positivo. De las siguientes, la actitud correcta a seguir es:

1. Solicitar un escoliograma en bipedestación y en función de los grados de Cobb decidir tratamiento con corsé.

2. Proponer una revisión en 6 meses con un escoliograma en decúbito.

3. Derivar a fisioterapia para estiramientos y masajes del raquis.

4. Recomendar natación para corregir y revisión en 6 meses.

Respuesta correcta: 1

Se trata de una escoliosis idiopática del adolescente, detectada mediante exploración clínica. Para decidir un tratamiento, hay que cuantificar el grado de curva y la madurez ósea, y según ello decidir el plan terapéutico.
Raramente se hacen radiografías en decúbito en escoliosis (a veces para valorar la corregibilidad de la curva en ausencia de gravedad), por lo que la opción 2 es falsa. 

Aunque está muy bien tener un raquis flexible y con buena musculatura estabilizadora, ni los masaje ni los estiramientos ni la natación corrigen la escoliosis, y de hecho en los tratados de pregrado ni se contempla.

De Martínez, F. M., & Martínez-Aedo, A. L. U. (2022). Traumatología y Ortopedia para el grado en medicina. Elsevier Health Sciences.; 

“Pruebas de imagen 

Para el diagnóstico inicial se recomienda realizar una tele­rradiografía posteroanterior en bipedestación (en caso de pacientes ambulantes) de la columna vertebral, así como

una telerradiografía lateral de columna. Las proyecciones anteroposteriores permiten una mejor definición de la columna, pero suponen una mayor absorción de radiación para la glándula mamaria, por lo que deben evitarse. El grado de desarrollo de la fisis de la cresta ilíaca según el test de Risser proporciona información sobre el potencial de crecimiento óseo del paciente (y de la curva escoliótica) y debe anotarse (fig. 37.5).”

“En las formas juvenil y del adolescente el tratamiento

depende de los grados de deformidad:

  • 10 – 20°: observación seriada.
  • 20-50°: corsés de termoplástico (Boston, Cheneau o Milwaukee, según dónde se localice el ápex de la curva principal).
  • 40-50°: con Risser menor de 3 estaría indicada la artrodesis quirúrgica; con Risser mayor o igual de 3 el potencial remanente de crecimiento de la curva es escaso y se puede valorar la observación por si tendiera a progresar en la edad adulta”

pregunta 100.

Mujer de 52 años, carnicera de profesión, sin antecedentes de interés ni caídas, que presenta dolor en el hombro de 4 meses de evolución al levantar el brazo. El dolor es de características inflamatorias y las maniobras de impingement positivas. Señale la actitud INCORRECTA:

1. Iniciar tratamiento con AINE, solicitar una ecografía y derivar a rehabilitación

2. Derivar a un cirujano especialista para hacer una artroscopia de hombro.

3. Si hay limitación pasiva del balance articular, con la sospecha de capsulitis adhesiva, derivar a rehabilitación urgente para movilizaciones pasivas.

4. Se sospecha una lesión del manguito de los rotadores (supraespinoso con bursitis subacromial), por lo que hay que potenciar la musculatura descoaptante del hombro.

Respuesta 2

No se recomienda DE ENTRADA el manejo quirúrgico de la omalgia sin completar estudio (no hay información sobre pruebas de imagen complementarias) y sin haber intentado razonablemente un manejo no quirúrgico con un tratamiento rehabilitador (sobre cuál ya se puede debatir, que serían las otras opciones, pero es que nos piden la INCORRECTA.

Cito Martínez, F. M., & Martínez-Aedo, A. L. U. (2022). Traumatología y Ortopedia para el grado en medicina. Elsevier Health Sciences. 

“TRATAMIENTO CONSERVADOR

Dada la naturaleza del dolor subacromial, el objetivo del tratamiento no quirúrgico será la desaparición de los síntomas, aunque los pacientes continúen con sus roturas tendinosas. La literatura científica informa de que aproximadamente en el 50% de los casos se consigue el objetivo, especialmente en aquellos sin una evolución prolongada previa. Este tratamiento conservador implica: modificaciones de las actividades que provocan dolor; antiinflamatorios no esteroideos (AINE); calor local; ejercicios de estiramiento y potenciación del manguito y de la musculatura paraescapular, y ocasionalmente infil­tración subacromial de una mezcla de anestésico local y corticoides de depósito, hasta un máximo de tres infil­traciones. Este tratamiento estará especialmente indica­do en pacientes con dolor sin debilidad significativa o

progresiva. Puede intentarse razonablemente durante un máximo de 3 meses antes de tomar otra determinación terapéutica.

TRATAMIENTO QUIRÚRGICO Como hemos visto, la presencia de una rotura tendinosa no implica necesariamente cirugía, aunque se prefiere indicarla precozmente en roturas completas en pacientes jóvenes con antecedente traumático. En otros casos la indicación vendría dada por el dolor y la presencia de un déficit funcional que no ha respondido a tratamiento conservador.”

pregunta 101.

Mujer de 56 años que consulta porque hace 4 meses se torció el tobillo en la playa y desde entonces no ha dejado de molestarle. Se cansa subiendo escaleras y le cuesta usar calzado plano. En la exploración presenta dolor en el seno del tarso, el talón y la cara medial del tobillo. Al examen podoscópico presenta el talón en valgo, aunque corrige de puntillas. La estructura más probablemente afecta es:

1. El tendón del músculo tibial posterior.

2. El ligamento peroneo-astragalino anterior.

3. El tendón del músculo flexor largo del dedo gordo.

4. El ligamento intertalocalcáneo.

Respuesta 1. 

Se trata de un pie plano del adulto (o colapso progresivo del arco del pie), que se debe a la disfunción del tibial posterior. Aparece típicamente en mujeres algo mayores, a menudo con algo de sobrepeso, y pueden recordar un desencadenante traumático aunque puede tratarse de un sesgo de memoria. 

Cito Martínez, F. M., & Martínez-Aedo, A. L. U. (2022). Traumatología y Ortopedia para el grado en medicina. Elsevier Health Sciences. 

PIE PLANO POR DISFUNCIÓN DEL MÚSCULO TIBIAL POSTERIOR

Es la causa más habitual de aparición de pie plano en el adulto. Suele ocurrir en mujeres de mediana edad, con vida sedentaria y sobrecarga ponderal. Generalmente existen trastornos endocrinos propios de la menopausia

Estadio 1. Caracterizado por la existencia de una paratendinitis en un tendón de longitud normal. La bóveda presenta un aspecto similar al del pie sano y la movilidad del complejo articular periastragalino es normal. Existe inflamación y dolor en el trayecto distal del tendón hasta su inserción en el escafoides. En la resonancia magnética (RM) puede detectarse el derrame de la vaina sinovial del tendón.

Estadio 2. El tendón está degenerado (tendinosis) y se hace progresivamente insuficiente, con lo que aparece el valgo de talón. Si exploramos el pie desde atrás, observaremos que, debido a la pronación del retropié y a la abducción del antepié, se ven más dedos en la parte externa del lado afecto (signo de «dema­siados dedos»). Si pedimos al paciente que se ponga de puntillas, en los pies normales el talón pasa de la posición de valgo a la de varo por la acción del mús­culo TP; a medida que se va haciendo insuficiente, no se producirá dicha corrección y el talón permanecerá en valgo; la estancia de puntillas sobre el pie afecto se hace imposible”

pregunta 102.

Cuidadora de personas mayores de 55 años, sin antecedentes de interés, diestra, que refiere dolor y acorchamiento en cara palmar de ambas manos, peor en la derecha, con predominio nocturno, así como sensación de gran hinchazón al despertarse por las mañanas, siendo algo más intenso en los dedos 2º y 3º. No refiere ningún otro síntoma en miembros superiores ni en cuello. ¿De las siguientes, cuál es la primera sospecha diagnóstica?:

1. Espondiloartrosis cervical C5-C6.

2. Compresión cubital epitroclear.

3. Hernia discal cervical C6-C7.

4. Síndrome del túnel carpiano.

Respuesta 4. 

Es un caso clínico típico de un síndrome del túnel carpiano, que no debería dar problemas a los candidatos MIR. 

Cito Martínez, F. M., & Martínez-Aedo, A. L. U. (2022). Traumatología y Ortopedia para el grado en medicina. Elsevier Health Sciences. 

“El caso más común es el de una mujer de mediana edad que presenta parestesias en los dedos segundo, tercero y cuarto. Por la noche, la sintomatología se exacerba y des­pierta a la paciente, debido a la sensación de quemazón y a las parestesias. Esta sintomatología mejora median­te sacudidas o masaje de la muñeca. Los dedos pueden hincharse, y el dolor y las parestesias pueden irradiarse proximalmente hasta el hombro. “

pregunta 103.

Acude de nuevo a urgencias una mujer de 67 años que ayer fue tratada de una fractura del extremo distal del radio izquierdo mediante reducción cerrada bajo anestesia focal y yeso antebraquial. Aqueja dolor insoportable, que aumenta cuando se le mueven los dedos; efectivamente es incapaz de movilizar activamente los dedos y estos están muy hinchados. ¿Qué complicación debemos tratar inmediatamente?:

1. Síndrome de dolor regional complejo.

2. Síndrome compartimental local.

3. Rotura tendinosa de extensores.

4. Desplazamiento secundario de la fractura.

Respuesta 2

El cuadro de dolor intenso que aumenta al movilizar los dedos en contexto de un enyesado (compresión externa) tras un traumatismo es típico, y es una de las complicaciones a vigilar en toda fractura inmovilizada con yeso. 

El dolor regional complejo (respuesta 1) tarda más en aparecer, la rotura tendinosa de los extensores (respuesta 2) no suele dar ese aumento del dolor (y aparece más tarde en el proceso de la fractura, por “roce” con el foco). El desplazamiento secundario (respuesta 4) no suele dar ese tipo de dolor ni impotencia funcional ni edema (salvo que no le hayan puesto una escayola siquiera y no esté inmovilizada la fractura, claro). 

Cito Martínez, F. M., & Martínez-Aedo, A. L. U. (2022). Traumatología y Ortopedia para el grado en medicina. Elsevier Health Sciences. 

CLÍNICA QUE DEBE HACER SOSPECHAR UN SÍNDROME COMPARTIMENTAL

• Dolor intenso que no cede con analgesia habitual.

• Dolor al estiramiento pasivo de los músculos.

• Aumento de tensión de los compartimentos a la palpación.

• Parestesias.

pregunta 104.

Paciente de 73 años activo, que presenta dolor de intensidad EVA 8-9 en el hombro derecho (miembro dominante) con pérdida de movilidad. Se realiza una radiografía y una RM que muestran una rotura completa del manguito rotador con ascenso de la cabeza humeral y erosión (acetabulización) del borde inferior del acromion. ¿Cuál de las siguientes es la actitud terapéutica más adecuada?:

1. Prótesis anatómica de hombro.

2. Infiltración del resto de manguito con células mesenquimales.

3. Reparación artroscópica del manguito rotador mediante anclajes en doble hilera.

4. Prótesis invertida de hombro.

Respuesta 4. 

Los hallazgos radiológicos sugieren una artropatía del manguito en su estadio final, con ascenso de la cabeza humeral, que roza con el borde inferior del acromion, en presencia de rotura completa crónica del manguito. Aquí no servirán las células mesenquimales (respuesta 2) ni tampoco se trata de una lesión reparable con sutura (opción 3), y la prótesis anatómica (opción 1) suele reservarse en los casos en los que se puede aprovechar el manguito. La prótesis invertida del hombro desciende y separa el centro de rotación de la cabeza humeral, cuya movilidad deja de depender de la presencia del manguito rotador, y es el estándar en la artropatía terminal del manguito. 

Cito Martínez, F. M., & Martínez-Aedo, A. L. U. (2022). Traumatología y Ortopedia para el grado en medicina. Elsevier Health Sciences. 

“Artroplastia: indicada en la artropatía de manguito o en lesiones irreparables muy sintomáticas. Hoy día utilizamos prótesis denominadas «invertidas» (concavidad en húmero y convexidad en glena) que utilizan la acción única del deltoides para la eleva­ción del brazo mediante un diseño semiconstreñido (fig. 20.2B). Sin embargo, su uso está restringido aún a personas añosas para evitar la desimplantación secundaria a su excesiva demanda”

pregunta 105.

Niña de 6 años que estaba celebrando un cumpleaños en un castillo hinchable, cayendo sobre el brazo extendido. Acude a urgencias con dolor y deformidad a nivel del húmero distal derecho e impotencia funcional del codo. A la exploración no se palpa pulso radial, el relleno capilar de los dedos es de 2 segundos y el color de la mano es sonrosado. La actitud más recomendable es:

1. Realizar una arteriografía de la arteria humeral.

2. Reducir y fijar la fractura en quirófano y observar la respuesta vascular.

3. Proceder a la exploración quirúrgica de la arteria humeral.

4. Hacer una ecografía Doppler de la arteria radial a nivel de la muñeca.

Respuesta 2. 

Se trata de una mano sin pulso bien perfundida, con un probable espasmo de la arteria humeral en el contexto del traumatismo y la deformidad de la fractura; la actitud es corregir la deformidad y fijar la fractura y vigilar la respuesta vascular, en la enorme mayoría de los casos se recupera el pulso espontáneamente. En el caso de una mano sin pulso isquémica (no sonrosada), se procederá a la valoración conjunta con cirugía vascular, sin demorar el tratamiento pendiente de la realización de pruebas complementarias. El eco-Doppler puede salir normal en niños en presencia de lesión de la arteria humeral, gracias a la gran circulación colateral que tienen los niños en el codo. 

De https://www.orthobullets.com/pediatrics/4007/supracondylar-fracture–pediatric?hideLeftMenu=true

“Vascular Injury

  • radial pulse absent on initial presentation in 7-12%
  • pulseless hand after closed reduction and pinning (3-4%) 
  •  if perfusion is lost following reduction and pinning, pins should be removed immediately 
  • decision to explore is based on quality of extremity perfusion rather than absence of pulse
  • arteriography is NOT indicated in isolated injuries
  • role of doppler is unclear and does not change treatment”

pregunta 106.

Neonato de 3 semanas que presenta un reflejo de Moro asimétrico. Parto vaginal de madre primípara, peso al nacer 4.600 g. Se palpa un bulto en la clavícula derecha y mantiene el hombro derecho en aducción, con el codo extendido y flacidez de muñeca y dedos de la mano derecha. En la radiografía de tórax realizada al nacer se observa una elevación del hemidiafragma derecho. Es signo de mal pronóstico la existencia de:

1. Ausencia de contracción del bíceps a esta edad.

2. Una fractura de clavícula ipsilateral.

3. Un síndrome de Horner ipsilateral.

4. Una luxación congénita de la cadera ipsilateral.

Respuesta 3

Describen un caso clínico típico de parálisis braquial obstétrica (con fractura de clavícula en consolidación asociada, de allí el bulto). La elevación del hemitórax derecho apunta a una lesión asociada del nervio frénico. La asociación de un síndrome de Horner (lesión de primera raíz torácica) sugeriría una lesión completa del plexo braquial, de peor pronóstico. 

La luxación congénita de cadera ipsilateral (opción 4) es una asociación frecuente, igual que la presencia de la fractura de clavícula ipsilateral (opción 2), pero no cambia el pronóstico. Que se recupere el biceps a los 3 MESES (no 3 semanas) es un signo de buen pronóstico (opción 1). 

Referencia: https://www.orthobullets.com/pediatrics/4117/obstetric-brachial-plexopathy-erbs-klumpkes-palsy?hideLeftMenu=true

PROGNOSIS

  • 90% of cases will resolve without intervention  spontaneous recovery may occur for up to 2 years
  • Prognostic variables for spontaneous recovery
    • favorable
      • Erb’s Palsy
      • complete recovery possible if biceps and deltoid are anti-gravity by 3 months
      • early twitch biceps activity suggests a favorable outcome 
      • return of elbow flexion by three months is associated with recovery in vast majority (up to 100%) of patients
    • Poor
      • lack of biceps function by 3 months 
      • preganglionic injuries (worst prognosis)
      • avulsions from the cord, which will not spontaneously recover motor function
      • loss of rhomboid function (dorsal scapular nerve)
      • elevated hemidiaphragm (phrenic nerve)
      • Horner’s syndrome (ptosis, miosis, anhydrosis)
      • less than 10% recover spontaneous motor function  
      • C7 involvement
      • Klumpke’s Palsy

pregunta 107.

Hombre de 62 años sin antecedentes de interés y profesión sedentaria, que consulta por dolor muy intenso, incapacitante, en región lumbar, de 1 semana de evolución, irradiado a nalga, cara posterior de muslo izquierdo y cara anterolateral de pierna izquierda. El signo de Lasègue es negativo y no presenta déficit de fuerza en las extremidades inferiores. La actitud terapéutica más adecuada, de entre las siguientes, es:

1. Remitir de forma urgente a neurocirugía para pruebas de imagen y valoración quirúrgica.

2. Realizar radiografía y resonancia magnética urgentes de raquis lumbar.

3. Pautar un antiinflamatorio no esteroideo a dosis alta, evitar el reposo en cama, calor local suave y revisión en 2 semanas.

4. Solicitar un electromiograma preferente de extremidades inferiores.

Respuesta 3

Pregunta de un cuadro de lumbociatalgia sin datos de alarma (red flags) que no debería dar problemas a los candidatos MIR. No está indicada ni la realización urgente de pruebas de imagen ni la derivación urgente a neurocirugía ni la realizaciónd e un electromiograma en ausencia de déficit motor. 

Cito Rodríguez, A., Domínguez, A., & Boyero, L. (2019). Minipildoras de consulta rápida: Manual para residentes y médicos de familia. Madrid: MEDGEN SA.

“Pruebas complementarias: 

Pruebas de imagen: 

  • Radiografía de columna lumbosacra: Sólo está indicada si existen síntomas de alarma
  • Gammagrafía ósea: Ante sospechas de neoplasias, osteomielitis, abscesos y fracturas patológicas
  • Mielografía, TC y RMN: Cuando la radiografía y la gammagrafía no aportan el diagnóstico, sobre todo si sospechamos discopatías o neoplasias”

Mucha suerte a todos y después de corregir, a descansar, que bien os lo habéis ganado

LAS RESPUESTAS TAMBIÉN SE PUBLICARÁN EN LOS DIFERENTES BLOGS/redes DE LOS PROFESIONALES QUE PARTICIPAN EN LA INICIATIVA.

Puedes comentar sobre este tema en el foro

OS PEDIMOS DESDE AQUÍ LA MÁXIMA DIFUSIÓN A LA INICIATIVA PARA QUE LLEGUE AL MÁXIMO DE OPOSITORES MIR PRESENTES Y FUTUROS PARA QUE LES SEA DE AYUDA.

Preguntas MIR enero 2023: Cirugía Ortopédica y Traumatología.

Respuestas comentadas a las preguntas de Traumatología del examen MIR 2022
Respuestas comentadas a las preguntas de Traumatología del examen MIR 2022

Respuestas cortesía de Aristides D oleo Maldonado ( @AristidesDoleo ), Cristina Ojeda Thies (@ojedathies), Antonio Sánchez Fernández (@AnSanFer_Dr) y Miguel Vázquez Gómez (@miguelvazquezdr)

Estas respuestas sólo reflejan la opinión de aquellos voluntarios que han elegido participar en la Iniciativa MIR 2.0 de respuesta al examen MIR de enero de 2023. (La numeración se corresponde con la Versión 0 del examen)

Imagen

3. Pregunta asociada a la imagen 3.

Mujer de 78 años, sin antecedentes médicos de interés, que presenta la fractura-luxación de húmero proximal mostrada en la imagen. ¿Cuál es el tratamiento más recomendable?:

1 Clavo trocantérico largo acerrojado proximal y distal.

2 Rehabilitación intensiva inmediata sin inmovilización.

3 Inmovilización con yeso braquial colgante de Caldwell.

4 Artroplastia glenohumeral

Solución:  4.

Comentario:

Lo importante es conseguir una reducción adecuada. En una paciente añosa es esperable mala calidad ósea por lo que puede terminar en artroplastia. Ahora bien, creo que de entrada lo que debes intentar es una reducción cerrada ,si esto no es posible, entonces una reducción abierta. Aquí puede acabar en osteosíntesis (con placa o clavo) o con artroplastia.

@ojedathies
Nº 3 – 4: Es una fractura-luxación de húmero proximal en paciente mayor, riesgo muy alto de necrosis avascular. La opción más correcta es la D (Artroplastia).
Ninguna otra opción menciona reducir la luxación, por lo que por eliminación también se puede adivinar.

Imagen

23. Pregunta asociada a la imagen 23.

Varón de 80 años, sin antecedentes de interés, que refiere dolor cervical progresivo e incapaci- tante que no cede con tratamiento analgésico. En la imagen de TC ¿cuál es el diagnóstico más probable y su posible origen?:

1 Espondilosis de origen degenerativo.

2 Espondilodiscitis de origen infeccioso.

3 Metástasis blásticas por un cáncer de próstata.

4 Metástasis líticas por un cáncer de pulmón.

Solución:          3

Comentario:

@ojedathies
Nº 23 – 3: Si miramos bien la imagen de la TC, se ven lesiones blásticas en varias vértebras. Aunque el paciente tenga también espondilosis degenerativa (tiene 80 años!!), la ausencia de mejora con analgesia, el empeoramiento y los hallazgos TC sugieren que es la opción 3.

Imagen

97. Varón de 52 años, diabético, que consulta por pie caído derecho indoloro. En la exploración hay debilidad del músculo tibial anterior y peroneos derechos, con reflejo rotuliano y aquíleos preser- vados. ¿Cuál es el diagnóstico más probable?:

1. Radiculopatía S1 derecha.

2. Mononeuropatía del ciático poplíteo externo derecho.

3. Radiculopatía L5 derecha.

4. Amiotrofia diabética.

Nº 97 – 2: Pie caído, afectación tibial ant. + peroneos con reflejos intactos -> pensar en lesión del CPE. Reflejo Aquileo intacto descarta la radiculopatía S1, quedando la radiculopatía L5 como duda. Para eso la pista es «indoloro» (en modo MIR). La amiotrofia da dolor+debilidad

Imagen

108. Varón de 55 años sin antecedentes de interés que tras realizar un esfuerzo nota dolor lumbar irradiado hacia el miembro inferior derecho por la parte anterolateral del muslo y cara anterior de la rodilla. Exploración: maniobra de Lasègue positiva a 40 º, reflejo rotuliano disminuido y dificultad para la marcha de talones. ¿Cuál es el diagnóstico más probable?:

1 Hernia discal L2-L3 derecha.

2 Hernia discal L3-L4 derecha.

3 Hernia discal L4-L5 derecha.

4 Hernia discal L5-S1 derecha.

Solución:

Comentario:

La sensibilidad de cara anterior de muslo corresponde a L4 así como el reflejo rotuliano también corresponde a esta raíz.

Marcha de talones difícil: implica debilidad del tibial anterior.

Nº 108 – 3: El dermatomo + reflejo rotuliano corresponde a raíz L4, y la debilidad de marcha de talones es normalmente la raíz L5 (puede ser L4). Apunta hacía hernia L4-L5 con afectación foraminal de la raíz L4 y paracentral de la L5.

Imagen

109. Acude a urgencias un paciente de 5 años con fiebre de 48 horas de evolución, con mala respuesta a los antitérmicos y rechazo al apoyo del miembro inferior izquierdo. En la exploración física se observa limitación de la movilidad de la cadera. En la analítica aumento de la PCR y leucocitosis. ¿Qué prueba complementaria se debe solicitar en primer lugar?:

1 Radiografía de cadera.

2 Ecografía de cadera.

3 RM de ambas caderas.

4 Artrografía de cadera.

Solución:

2 ecografía.

Comentario:

Creo que podría impugnarse dado que en un servicio de urgencias se pedirían ambas. Aquí hay que sospechar una artritis séptica de cadera y una ecografía vería un aumento del líquido articular en la cadera. En un primer momento la radiografía no aportaría información.

Nº 109 – 2: Cadera del niño = clásica MIR. Edad, tiempo evolución, fiebre, leucos, PCR… pensar artritis séptica. No hay pistas de sinovitis transitoria o Perthes. La Rx sería normal (pero se hace en urgencias) y lo que da el diagnóstico es la eco de cadera (y punción guiada).

Imagen

110. Mujer de 82 años con antecedentes de insuficiencia cardíaca congestiva y cáncer de mama que consulta por una fractura de muñeca izquierda. La fractura es estable, no es conminuta ni presenta trazos intraarticulares. ¿Cuál de los siguientes tratamientos es el más adecuado?:

1 Reducción cerrada y yeso antebraquial.

2 Artroplastia total de muñeca.

3 Osteosíntesis con placas dorsal y volar.

4 No inmovilizar, rehabilitación inmediata.

Solución:1

Comentario:

Nos habla de una paciente con un perfil más acorde a tratamientos no quirúrgicos y nos hablan de una lesión no desplazada, estable, no conminuta y sin afectación articular que tampoco sería quirúrgica en un paciente de mejor perfil, en general.

Nº 110 – 1: Colles = clásico del MIR. Todos los elementos son favorables al tratamiento conservador con reducción cerrada + yeso antebraquial (estable, sin conminución, extraarticular, paciente mayor…).
No debería dar problemas a los candidatos.

Imagen

111. Empleada de hogar de 35 años, sin antecedentes de interés, diestra, que refiere dolor en 5º dedo de mano izquierda y en cara interna de antebrazo izquierdo acompañado de acorchamiento y pérdida parcial de sensibilidad. ¿Cuál sería la primera sospecha diagnóstica?:

1 Síndrome del túnel carpiano.

2 Hernia discal cervical C3-C4.

3 Compresión cubital epitroclear.

4 Tendinitis de De Quervain.

Solución:3

Comentario:

Sintomatología cubital desde antebrazo.

Nº 111 – 3: Dolor 5º dedo + cara medial antebrazo, y pérdida de sensibilidad, es sugestiva de compresión del n. cubital en el canal epitroclear (muy común en empleadas domésticas). La duda sería una radiculopatía C8 pero no dan la opción.

Imagen

112. Respecto a la pseudoartrosis del escafoides, señale la afirmación INCORRECTA:

1 Suele cursar con una necrosis del fragmento distal de la fractura.

2 Puede cursar de manera asintomática.

3 A largo plazo suele provocar una artrosis radiocarpiana.

4 En estadios iniciales el tratamiento de elección es la fijación de la fractura y aporte de injerto óseo.

Respuesta:

1, es proximal

Comentario: Nº 112 – 1: Otra pregunta muy asequible. Machacado en la carrera y academias; la arteria entra al escafoides por el polo distal y el que tiene la circulación precaria es el proximal. Adjunto dibujo de la arteria. Las demás son ciertas.

Imagen

Imagen

113, Varón de 69 años, con índice de masa corporal 37 y gran obesidad central (abdominal) que aqueja dolor lumbar incapacitante que se extiende hasta ambas regiones glúteas, pero no irradia a miembros inferiores. Le impide permanecer de pie quieto y caminar más de 10 minutos, pero desaparece sentado y en la cama. La maniobra de Lasègue es negativa. ¿Cuál es el diagnóstico más probable?:

1 Hernia discal a nivel L5-S1.

2 Estenosis de canal lumbar.

3 Hernia discal a nivel L4-L5.

4 Espondilodiscitis bacteriana.

Solución: 2

Comentario: Nº 113 – 2. Todos los datos (obesidad abdominal, dolor glúteo, no afectación MMII, Lasègue normal, mejoría sentado con claudicación al caminar…) apuntan a estenosis del canal con claudicación neurogénica. No hay datos ni de hernia discal ni de espondilodiscitis.

Imagen

114. Ante una lesión diafisaria de húmero de carácter lítico que afecta a la totalidad del diámetro del hueso en un varón de 67 años con el antecedente de un cáncer de pulmón (supervivencia estimada superior a 2 años) y que le provoca dolor continuo mal controlado. ¿Cuál de las siguientes es la mejor actitud terapéutica?:

1 Actitud expectante, ya que se trata de un hueso sin carga, lo que reduce el riesgo de fractura.

2 Aplicación de mórficos a dosis altas para controlar el dolor que le permitan realizar fisioterapia para recuperar la función.

3 Ajustar un yeso de forma correcta para disminuir el riesgo de fractura.

4 Colocación de un clavo endomedular acerrojado.

Respuesta: 4

Comentario:

Para calidad de vida, procedimiento poco agresivo para mejorar el dolor.

Escala de Mirels: Dolor continuo mal controlado: 3 puntos Litica: 3 puntos Todo el diametro del hueso: 3 puntos MMSS: 1 punto 9 o mas puntos: fijar

Nº 114 – 4: La escala de Mirels ha caído varias veces en el MIR. Todos los datos son 3 puntos (dolor importante, todo el diámetro, lítico) salvo que esté en miembro superior (1 punto). 9 o más puntos: enclavado profiláctico. Con 2 años de supervivencia el yeso es una putada.

Imagen

Imagen

117. Mujer de 81 años asmática que utiliza los corticoides de forma frecuente para controlar los brotes de broncoespasmo y que consulta porque tiene un dolor agudo intenso en la región dorso- lumbar después de caer en casa desde una silla. ¿Cuál de las siguientes es la actitud más recomendable a seguir?:

1 Solicitar de forma urgente radiografías de columna dorsal y lumbar y pautar tratamiento analgésico para el dolor.

2 Solicitar una RM de columna dorsal y lumbar y pautar tratamiento analgésico y reposo para el dolor.

3 Pautar tratamiento analgésico y empezar tratamiento antirreabsortivo.

4 Solicitar una densitometría, pautar tratamiento analgésico y empezar con tratamiento antireabsortivo, calcio y vitamina D.

Respuesta: 1

Comentario: Nº 117 – 1. Mayor, corticoterapia, dolor raquideo intenso tras caída -> pensar en fractura osteoporótica, se confirma con Rx. La RM si hay sospecha con Rx normal. Ek tratamiento de la osteoporosis toca también, pero predomina el diagnóstico y la analgesia.

Mucha suerte a todos y después de corregir, a descansar, que bien os lo habéis ganado

LAS RESPUESTAS TAMBIÉN SE PUBLICARÁN EN LOS DIFERENTES BLOGS DE LOS PROFESIONALES QUE PARTICIPAN EN LA INICIATIVA.

Puedes comentar sobre este tema en el foro

OS PEDIMOS DESDE AQUÍ LA MÁXIMA DIFUSIÓN A LA INICIATIVA PARA QUE LLEGUE AL MÁXIMO DE OPOSITORES MIR PRESENTES Y FUTUROS PARA QUE LES SEA DE AYUDA.

2017-04 Fractura supracondílea

Cada profesión, creo, otorga una visión propia de la vida, incluso de las situaciones cotidianas. Cuando te dedicas a la traumatología y ves un parque con niños jugando, uno de esos con estructuras para que se suban a jugar, nos viene a la mente lo que me encargaron ilustrar hace unos años: las fracturas supracondíleas. 

Lo que querían del dibujo era que sirviera de apoyo gráfico a una charla para hablar de fracturas supracondíleas. 

Imagen

 

¿Os ha gustado?

Preguntas MIR 2022: Cirugía Ortopédica y Traumatología.

Imagen

Respuestas cortesía de Aristides D oleo Maldonado ( @AristidesDoleo ), Cristina Ojeda Thies (@ojedathies), Antonio Sánchez Fernández (@AnSanFer_Dr) y Miguel Vázquez (@miguelvazquezdr)

Estas respuestas no son las oficiales ( las actualizaremos con las mismas en cuanto salgan) sólo reflejan la opinión de aquellos voluntarios que han elegido participar en la Iniciativa MIR 2.0 de respuesta al examen MIR de enero de 2022. (La numeración se corresponde con la Versión 0 del examen)

EDITO: Ya han salido la plantilla de respuestas oficiales del ministerio y coinciden plenamente con las respuestas que dimos al día siguiente del examen.

https://traumatopedia.com/mir/preguntas-traumatologia-mir-2022/

Pregunta 110

El osteosarcoma es un tumor óseo maligno que:

1. Es más frecuente en la primera década de la vida.

2. Su tratamiento con quimioterapia y radioterapia suele evitar la cirugía.

3. Afecta más frecuentemente a huesos en la proximidad de la articulación de la rodilla.

4. Si presenta enfermedad de Paget previa tiene un mejor pronóstico.

Respuesta correcta: 3

Comentario @AristidesDoleo

El osteosarcoma es el tumor óseo maligno mas frecuente en la segunda década de la vida, el tratamiento consiste en quimioterapia preoperatoria durante 8-12 semanas + cirugía de resección tumoral y posteriormente quimioterapia de mantenimiento durante 6-12 meses, su localización mas frecuente es el femur distal y la tibia proximal, otros sitios comunes de localización incluye el humero proximal, fémur proximal y pelvis. Una de las complicaciones de la enfermedad de Pagets es un aumento significativo en la incidencia de osteosarcoma, alrededor de 1% de estos pacientes desarrollan osteosarcoma, este riesgo contribuye significativamente a la mortalidad y morbilidad.

Comentario @ojedathies, @AnSanFer_Dr y @miguelvazquezdr

El osteosarcoma presenta mayor incidencia en la segunda década de la vida. Respuesta 1 incorrecta.

El tratamiento multimodal (cirugía y quimioterapia), ha conseguido que dos tercios de los pacientes que no presentan metástasis en el diagnóstico superen la enfermedad. Además, un tercio de los que presentan metástasis logran un aumento considerable de la supervivencia sin recidivas. Respuesta 2 incorrecta.

El osteosarcoma ocurre generalmente en los huesos largos de las extremidades cerca de las metáfisis (la zona donde se produce el crecimiento). Según datos de Ottaviani G , los sitios más comunes son el osteosarcoma en fémur (42%, con 75% de los tumores en el fémur distal, es decir, el extremo más cercano a la rodilla), la tibia (19%, con el 80% de los tumores en la tibia proximal, es decir el extremo más cercano a la rodilla) y el húmero (el hueso del brazo) (10%). Respuesta 3 correcta.

El osteosarcoma es una complicación poco frecuente de la enfermedad ósea de Paget y su diagnóstico precoz es importante debido a su pronóstico altamente letal. Respuesta 4 incorrecta.

-DAHLIN, David C. Pathology of osteosarcoma. Clinical orthopaedics and related research, 1975, no 111, p. 23-32.

-TOMPSON, Lester DR. Osteosarcoma. Ear, Nose & Throat Journal, 2013, vol. 92, no 7, p. 288-290.

Pregunta 111

El síndrome del túnel carpiano:

1. Es más frecuente en varones.

2. Cursa con hipoestesia en el pulpejo de los dedos segundo a quinto.

3. Los síntomas se exacerban durante la noche.

4. Se acompaña de atrofia de la musculatura de la eminencia hipotenar.

Respuesta correcta: 3

Comentario @AristidesDoleo

El síndrome del túnel carpiano es una neuropatía compresiva del nervio mediano que afecta con mas frecuencia al sexo femenino, cursa con parestesias en 1-2 y borde radial del 3 dedo, con empeoramiento nocturno y atrofia de la musculatura tenar.

Comentario @ojedathies, @AnSanFer_Dr y @miguelvazquezdr

Respuesta correcta 3: El síndrome del túnel carpiano es más frecuente en mujeres (respuesta 1 incorrecta). La clínica neurológica habitual afecta al primer dedo, segundo, tercero y a la cara radial del cuarto dedo de la mano (respuesta 2 incorrecta). Generalmente el dolor y las parestesias despiertan al paciente por la noche (respuesta 3 correcta). La compresión nerviosa puede causar degeneración de la rama motora, lo que causa pérdida de fuerza y la atrofia de los músculos de la eminencia tenar (respuesta 4 incorrecta).

-KATZ, Jeffrey N.; SIMMONS, Barry P. Carpal tunnel syndrome. New England Journal of Medicine, 2002, vol. 346, no 23, p. 1807-1812.

-DE KROM, MCTFM, et al. Risk factors for carpal tunnel syndrome. American journal of epidemiology, 1990, vol. 132, no 6, p. 1102-1110.

-STEVENS, J. CLARKE, et al. Conditions associated with carpal tunnel syndrome. En Mayo Clinic Proceedings. Elsevier, 1992. p. 541-548.

Pregunta 112

Mujer de 61 años, administrativa, con antecedentes de sobrepeso, hipertensión, dislipemia y síndrome metabólico, que consulta por dolor en ambas nalgas, región trocantérea izquierda, cara lateral del muslo izquierdo hasta la rodilla y pierna izquierda hasta el tercio medio. El dolor aparece cuando levantamos el miembro inferior con la rodilla extendida, pero se alivia al flexionar la rodilla. ¿Cuál es la primera sospecha clínica?:

1. Artritis gotosa de cadera izquierda.

2. Artrosis coxofemoral izquierda.

3. Lumbalgia irradiada / lumbociatalgia.

4. Claudicación por estenosis de canal.

Respuesta correcta: 3

Comentario @AristidesDoleo

Signo de Lasegue positivo, reaparición de la clínica al ejecutar la maniobra de extensión del miembro afectado, compatible con afectación de raíces nerviosas a nivel de columna lumbosacra.

Comentario @ojedathies, @AnSanFer_Dr y @miguelvazquezdr

La sintomatología más común de un paciente con coxartrosis incluye: Dolor de cadera que limita la deambulación, molestias nocturnas e incluso en reposo, rigidez de cadera y bloqueo. La artrosis coxo-femoral cursa con un test de Lasegue negativo. Respuesta 2 incorrecta.

​​El dolor lumbar de carácter neuropático suele cursar con radiculopatía unilateral y distribuida según dermatomas. Los factores de riesgo reconocidos son con mayor frecuencia: mujer, obesidad, fumador y sedentarismo o situaciones de sedestación prolongada. La lumbociatálgia se suele referir en la nalga, cara posterior de los muslos e incluso inguinal. El caso clínico describe la maniobra o test de Lasegue; una prueba de provocación que evidencia irritación radicular en la región lumbosacra (Respuesta 3 correcta).

La claudicación por estenosis de canal cursa característicamente con dolor que empeora con la extensión del tronco (caminar, estar de pie) y se alivia con la flexión (sentarse, dormir en posición fetal…) Respuesta 4 incorrecta.

-SWEZEY, Robert L. Overdiagnosed sciatica and stenosis, underdiagnosed hip arthritis. Orthopedics, 2003, vol. 26, no 2, p. 173-174.

-TIBOR, Lisa M.; SEKIYA, Jon K. Differential diagnosis of pain around the hip joint. Arthroscopy: The Journal of Arthroscopic & Related Surgery, 2008, vol. 24, no 12, p. 1407-1421.

-POULTSIDES, Lazaros A.; BEDI, Asheesh; KELLY, Bryan T. An algorithmic approach to mechanical hip pain. HSS Journal®, 2012, vol. 8, no 3, p. 213-224.

-WILSON, John J.; FURUKAWA, Masaru. Evaluation of the patient with hip pain. American family physician, 2014, vol. 89, no 1, p. 27-34.

Pregunta 113

Varón de 35 años, cartero, sin antecedentes de interés, que acude a urgencias por un dolor agudo cervical de 24 horas de evolución, sin traumatismo previo, que irradia a brazo izquierdo hasta la mano y se acompaña de parestesias en el borde radial del antebrazo. No presenta pérdida de fuerza objetivable, conserva la movilidad del cuello aunque es dolorosa y se aprecia contractura de la musculatura paravertebral. La primera actitud será:

1. Tratamiento conservador con antiinflamatorios no esteroideos, calor local y reposo relativo.

2. Llamada urgente al neurocirujano para valoración quirúrgica.

3. Solicitud preferente de resonancia magnética y electromiograma.

4. Derivación preferente a consultas externas de traumatología.

Respuesta correcta : 1

Comentario @AristidesDoleo: Cervicalgia sin signos de alarma. Tratamiento conservador con un éxito entre el 75%-90%.

Comentario @ojedathies, @AnSanFer_Dr y @miguelvazquezdr

Respuesta 1 correcta. El 75% de los pacientes con radiculopatía mejoran con el tratamiento no quirúrgico, siendo este de elección en fases iniciales.

La resonancia magnética es de elección ante determinadas «red flags»: Fiebre, pérdida de peso, dolor nocturno, persistencia de la sintomatología a pesar del tratamiento conservador y pérdida de fuerza (respuesta 3 incorrecta).

-CARETTE, Simon; FEHLINGS, Michael G. Cervical radiculopathy. New England Journal of Medicine, 2005, vol. 353, no 4, p. 392-399.

-POLSTON, David W. Cervical radiculopathy. Neurologic clinics, 2007, vol. 25, no 2, p. 373-385.

-CARIDI, John M.; PUMBERGER, Matthias; HUGHES, Alexander P. Cervical radiculopathy: a review. HSS Journal®, 2011, vol. 7, no 3, p. 265-272.

Pregunta 114

Respecto a la rotura del ligamento cruzado anterior de la rodilla, señale la respuesta correcta:

1. Es frecuente que se produzca por un traumatismo sobre la cara anterior de la tibia con la rodilla en flexión.

2. Raramente cursa con hemartros.

3. La maniobra de Lachman es negativa.

4. El tratamiento con fisioterapia con potenciación del cuádriceps y los isquiotibiales puede evitar la cirugía.

Respuesta correcta: 4

Comentario @AristidesDoleo:

Rotura LCA. Traslación de la tibia anterior mientras la rodilla está en ligera flexión y valgo. La opción 1 hace referencia a la rotura del ligamento cruzado posterior. La rotura del LCA cursa con hemartros y signo de Lachman positivo. Una opción valida es el tratamiento conservador con buenos resultados en pacientes con baja demanda funcional.

Comentario @ojedathies, @AnSanFer_Dr y @miguelvazquezdr

1.Un traumatismo en cara anterior de la tibia produce una traslación de la misma a posterior. El ligamento que se tensaría en esa situación en primer lugar, y el que tendría más riesgo de dañarse por este mecanismo, es el ligamento cruzado posterior (el traumatismo está generando un “cajón posterior”).

“Se ha comprobado que el LCP representa el 95% de la fuerza total que se opone a la traslación posterior de la tibia sobre el fémur” (…) “Las lesiones del LCP son menos frecuentes que las del LCA y, por lo general, son consecuencia de hiperextensión o de traumatismos sobre la cara anterior de la tibia con la rodilla flexionada”.

Henry D. Clarke, W. Norman Scott, John N. Insall, Henrik B. Pedersen, Kevin R. Math, Vincent J. Vigorita, Fred D. Cushner. Capítulo 1: Anatomía. Insall & Scott Rodilla. Versión en español de la 4ª edición de la obra original en inlgés “Surgery of the Knee”. Madrid: 2007 Elsevier España S.A. Página 3-66.

El texto citado aparece en la página 37.

2.Las lesiones del ligamento cruzado anterior producen un hermartros importante y rápido.

Gürpınar T, Polat B, Polat AE, Çarkçı E, Öztürkmen Y. Diagnostic Accuracy of Lever Sign Test in Acute, Chronic, and Postreconstructive ACL Injuries. Biomed Res Int. 2019 Jun 9;2019:3639693. doi: 10.1155/2019/3639693. PMID: 31281835; PMCID: PMC6590604.

3.La maniobra de Lanchman ejerce una fuerza anteroposterior sobre la tibia proximal poniendo a prueba los ligamentos cruzados. Cuando el cruzado anterior está roto se produce una traslación a anterior de la tibia que detectaríamos con el test de Lanchman.

“Prueba de Lachman. Esta prueba clínica se utiliza para evaluar la laxitud y resistencia anterior con la rodilla en unos 20º de flexión. En esta posición se aplica un cajón anterior a la zona proximal de la pantorrilla y el explorador percibe el desplazamiento de la tibia y el tope de resistencia. Se considera anormal el más mínimo aumento del desplazamiento anterior de la tibia en comparación con la otra rodilla. También hay que describir con precisión el tope de resistencia”.

Craig S. Radnay, Giles R. Scuderi y W. Norman Scott. Capítulo 34: Clasificación de las lesiones de los ligamentos de la rodilla. Insall & Scott Rodilla. Versión en español de la 4ª edición de la obra original en inlgés “Surgery of the Knee”. Madrid: 2007 Elsevier España S.A. Página 537-559.

El texto citado aparece en las páginas 545-546.

4.Tanto el cuádriceps como los isquiotibiales son agonistas de la función del cruzado anterior (su contracción generan un vector de tracción hacia posterior de la tibia proximal – es por esto que en la cirugía protésica de rodilla una dehiscencia del abordaje genera una luxación con la tibia hacia anterior-) por lo que su potenciación pueden compensar la pérdida del cruzado anterior.

“La estabilidad aumenta gracias a los estabilizadores dinámicos, como los músculos que ejercen fuerza a través de la articulación de la rodilla” (en referencia al ligamento cruzado anterior).

Henry D. Clarke, W. Norman Scott, John N. Insall, Henrik B. Pedersen, Kevin R. Math, Vincent J. Vigorita, Fred D. Cushner. Capítulo 1: Anatomía. Insall & Scott Rodilla. Versión en español de la 4ª edición de la obra original en inlgés “Surgery of the Knee”. Madrid: 2007 Elsevier España S.A. Página 3-66.

El texto citado aparece en la página 36.

Pregunta 115

Varón de 27 años, deportista habitual, que refiere dolor en la pierna derecha tras la práctica de carrera continua. Ha acudido en varias ocasiones a un fisioterapeuta, siendo diagnosticado de una sobrecarga en gemelos. Han transcurrido varios meses, no ha mejorado y refiere dolor intenso tras la actividad física que cede con el reposo en las siguientes horas del ejercicio. ¿Qué prueba puede ayudar en el diagnóstico?:

1. Tomografía de emisión de positrones con 18 FDG.

2. Determinación de la presión del compartimento posterior inmediatamente tras actividad.

3. Ecografía-Doppler para descartar un trastorno circulatorio de la extremidad inferior.

4. Espectrometría por resonancia magnética.

Respuesta correcta: 2

Comentario @AristidesDoleo: Sospecha clínica de síndrome compartimental crónico (SCC). El diagnostico se realiza mediante la obtención de presiones compartimentales en reposo, durante el ejercicio y post ejercicio.

Comentario @ojedathies, @AnSanFer_Dr y @miguelvazquezdr

Respuesta 2 correcta: El síndrome compartimental de esfuerzo es una afección de la pierna inducida por el ejercicio. Se caracteriza por la isquemia reversible de los músculos de un compartimento muscular. El diagnóstico se realiza mediante la medición de las presiones compartimentales en reposo, durante el ejercicio y después del mismo (respuesta 2 correcta). El tratamiento suele consistir en fasciotomías de los compartimentos afectados. Aunque la ​​resonancia magnética no es muy útil para establecer el diagnóstico, puede ayudar en el diagnóstico diferencial.

-BLACKMAN, Paul G. A review of chronic exertional compartment syndrome in the lower leg. Medicine and science in sports and exercise, 2000, vol. 32, no 3 Suppl, p. S4-10.

-RORABECK, C. H.; BOURNE, R. B.; FOWLER, P. J. The surgical treatment of exertional compartment syndrome in athletes. JBJS, 1983, vol. 65, no 9, p. 1245-1251.

Pregunta 116

Mujer de 95 años que vive en una residencia, independiente para sus actividades básicas de la vida diaria, sale al jardín a pasear. Tiene antecedentes de HTA, dislipidemia, osteoporosis y deterioro cognitivo leve. Sufre una caída al levantarse por la noche al baño. La radiografía muestra una fractura subcapital desplazada de cadera derecha. ¿Cuál es el tratamiento recomendado?:

1. Hemiartroplastia de cadera.

2. Fijación con clavo trocantérico.

3. Fijación con tornillos canulados.

4. Conservador: vida cama-sillón.

Respuesta correcta: 1

Comentario. Fractura de cadera desplazada, Garden III-IV. Tratamiento quirúrgico. Hemiartroplastia de cadera. Opción 2 tratamiento para fracturas pertrocantereas. Opción 3. Fracturas no desplazadas. Opción 4. Pacientes no deambulantes con mínimo dolor y alto riesgo quirúrgico.

Pregunta 117

¿En relación con las fracturas de la epitróclea en los niños señale la afirmación INCORRECTA?:

1. Se desencadenan por un mecanismo de valgo forzado del codo en la caída.

2. Están asociadas hasta en un 50 % de los casos con una luxación postero-lateral del codo.

3. Pueden presentar asociada una neurapraxia del nervio cubital.

4. Es más frecuente en niños de edades entre 4 y 6 años.

Respuesta correcta: 4

  1. Mecanismo indirecto: una caída sobre la mano extendida que ocasiona una fuerza en valgo sobre el codo produce una lesión por avulsión, con un trazo de fractura que comienza en la metáfisis y se propaga en dirección distal a través de la superficie articular” Texto en relación a las epifisiólisis del cóndilo medial. RESPUESTA 1 CORRECTA
  2. La epifisiólisis del cóndilo medila se asocia con frecuencia a una luxación del codo, en general posterolateral; la luxación del codo es sumamente rara antes de que comience la osificación de la epífisis del cóndilo medial”. RESPUESTA 2 CORRECTA
  3. Es importante realizar una minuciosa exploración neurovascular porque puede haber síntomas de la lesión del nervio cubital”. RESPUESTA 3 CORRECTA
  4. La edad de presentación típica oscila entre los 8 y 14 años”. RESPUESTA 4 INCORRECTA

Bibliografía: 

Kenneth A. Egol, Kenneth J. Koval, Joseph D. Zuckerman. Capítulo 44 El codo pediátrico. Editor de sección: Norman Otsuka, M.D. Manual de fracturas. 5ª edición. Barcelona (España). Editorial Wolters Kluwer, 2015.Página 583-626. 

Los textos citados aparecen en la páginas 595-596.

Pregunta 118

Varón de 41 años que consulta por gonalgia de varios días de evolución. En la exploración se realiza el test de Thessaly (dolor con los movimientos de rotación interna y externa con la rodilla flexionada) que resulta positivo. ¿Cuál de las siguientes lesiones es más probable?:

1. Lesión meniscal.

2. Lesión por rotura del ligamento cruzado anterior.

3. Lesión por rotura del ligamento cruzado posterior.

4. Lesión por artropatía degenerativa.

Respuesta correcta: 1

  1. El test de Thessaly es una maniobra de exploración meniscal. El enunciado se centra exclusivamente en esta parte de la exploración y nos explica, además, que efectivamente es positiva lo que hace sospechar que existe lesión meniscal.
  2. No nos describen en ningún momento maniobras de exploración del ligamento cruzado anterior, tampoco nos hablan de un traumatismo que lo haya podido lesionar. 
  3. No nos describen en ningún momento maniobras de exploración del ligamento cruzado posterior, tampoco nos hablan de un traumatismo que lo haya podido lesionar.
  4. Aunque podría darse no es lo esperable en una persona de 41 años con una clínica que se ha establecido de forma aguda. 

The Thessaly test is a dynamic reproduction of load transmission in the knee joint and is performed at 5° and 20° of flexion.   The examiner supports the patient by holding his or her outstretched hands while the patient stands flatfooted on the floor. The patient then rotates his or her knee and body, internally and externally, three times, keeping the knee in slight flexion (5°). Then the same procedure is carried out with the knee flexed at 20°. Patients with suspected meniscal tears experience medial or lateral joint-line discomfort and may have a sense of locking or catching. The theory behind the test is that, with this maneuver, the knee with a meniscal tear is subjected to excessive loading conditions and almost certainly will have the same symptoms that the patient reported. The test is always performed first on the normal knee so that the patient may be trained, especially with regard to how to keep the knee in 5° and then in 20° of flexion and how to recognize, by comparison, a possible positive result in the symptomatic knee.

Karachalios, Theofilos (2005). Diagnostic Accuracy of a New Clinical Test (the Thessaly Test) for Early Detection of Meniscal Tears. The Journal of Bone and Joint Surgery (American), 87(5), 955–. doi:10.2106/JBJS.D.02338 

Pregunta 119

Mujer de 13 años, sin antecedentes relevantes, con menarquia hace 3 meses, seguida desde los 10 años por escoliosis idiopática que ha empeorado. En la exploración física presenta una giba de 7 grados en el test de Adams y en el escoliograma una curva toracolumbar T4-L1 de 35 grados de Cobb y un Risser 0. La actitud correcta a tomar será:

1. Recomendar natación y revisión en tres meses.

2. Prescribir una ortesis tipo corsé.

3. Derivar a fisioterapia para elastificación del raquis.

4. Revisar en 6 meses con una nueva radiografía.

Respuesta correcta: 2

Comentario: Ortesis. Angulo de Cobb entre 25º – 45º. Esqueleto inmaduro (Risser 0)

Se puede plantear tratamiento con ortesis en Risser de 0 a 2 dado que aún queda crecimiento por darse y puede producirse la corrección. 

Su uso se plantea para curvas entre 30-40º.

Imagen

  1. Choudhry MN, Ahmad Z, Verma R. Adolescent Idiopathic Scoliosis. Open Orthop J. 2016 May 30;10:143-54.
  2. Negrini S, Donzelli S, Aulisa AG, Czaprowski D, Schreiber S, de Mauroy JC, Diers H, Grivas TB, Knott P, Kotwicki T, Lebel A, Marti C, Maruyama T, O’Brien J, Price N, Parent E, Rigo M, Romano M, Stikeleather L, Wynne J, Zaina F. 2016 SOSORT guidelines: orthopaedic and rehabilitation treatment of idiopathic scoliosis during growth. Scoliosis Spinal Disord. 2018 Jan 10;13:3.
  3. Konieczny MR, Senyurt H, Krauspe R. Epidemiology of adolescent idiopathic scoliosis. J Child Orthop. 2013 Feb;7(1):3-9.
  4. Grauers A, Einarsdottir E, Gerdhem P. Genetics and pathogenesis of idiopathic scoliosis. Scoliosis Spinal Disord. 2016 Nov 28;11:45.
  5. Brink RC, Schlösser TPC, van Stralen M, Vincken KL, Kruyt MC, Hui SCN, Viergever MA, Chu WCW, Cheng JCY, Castelein RM. Anterior-posterior length discrepancy of the spinal column in adolescent idiopathic scoliosis-a 3D CT study. Spine J. 2018 Dec;18(12):2259-65.
  6. Stokes IA, Mente PL, Iatridis JC, Farnum CE, Aronsson DD. Enlargement of growth plate chondrocytes modulated by sustained mechanical loading. J Bone Joint Surg Am. 2002 Oct;84(10):1842-8.
  7. Stokes IA, Burwell RG, Dangerfield PH; IBSE. Biomechanical spinal growth modulation and progressive adolescent scoliosis–a test of the ‘vicious cycle’ pathogenetic hypothesis: summary of an electronic focus group debate of the IBSE. Scoliosis. 2006 Oct 18;1:16.
  8. Balioglu MB, Aydin C, Kargin D, Albayrak A, Atici Y, Tas SK, Kaygusuz MA. Vitamin-D measurement in patients with adolescent idiopathic scoliosis. J Pediatr Orthop B. 2017 Jan;26(1):48-52.
  9. Altaf F, Gibson A, Dannawi Z, Noordeen H. Adolescent idiopathic scoliosis. BMJ. 2013 Apr 30;346:f2508.
  10. Burton MS. Diagnosis and treatment of adolescent idiopathic scoliosis. Pediatr Ann. 2013 Nov;42(11):22

Pregunta 120

Sobre la fascitis plantar, señale la respuesta correcta:

1. El dolor mejora al andar de puntillas.

2. Se asocia a la presencia de espolón calcáneo.

3. El tratamiento de elección es quirúrgico.

4. Provoca dolor en la zona del talón, más intenso al empezar a caminar.

Respuesta correcta: 4

Respuesta correcta: 4. La fascitis plantar supone la inflamación de la aponeurosis de la fascia plantar en su origen en el calcáneo. La talalgia tiene un inicio gradual, más agudo al comenzar a caminar por las mañanas, y con un alivio progresivo a lo largo del día (respuesta 4 correcta). El dolor empeora al andar de puntillas (respuesta 1 incorrecta) y la primera línea de tratamiento es conservadora (respuesta 3 incorrecta). 

  • LEAGUE, Alan C. Current concepts review: plantar fasciitis. Foot & ankle international, 2008, vol. 29, no 3, p. 358-366.
  • GOFF, James D.; CRAWFORD, Robert. Diagnosis and treatment of plantar fasciitis. American family physician, 2011, vol. 84, no 6, p. 676-682.
  • SCHEPSIS, Anthony A.; LEACH, Robert E.; GORZYCA, J. Plantar fasciitis. Etiology, treatment, surgical results, and review of the literature. Clinical Orthopaedics and Related Research, 1991, no 266, p. 185-196.

Pregunta 121

Mujer de 79 años que ingresa por una fractura osteoporótica de cadera. Respecto a la prevención secundaria de las fracturas por fragilidad, señale la respuesta INCORRECTA:

1. La baja adherencia al tratamiento se asocia a un incremento del riesgo de fractura.

2. Los marcadores de remodelado óseo pueden ser útiles para monitorizar de forma precoz la respuesta al tratamiento.

3. La vitamina D en monoterapia es eficaz en la reducción de dichas fracturas en personas mayores no institucionalizadas.

4. El aumento de calcio dietético o tomar suplementos de calcio de forma aislada no protegen frente a la aparición de fracturas.

Respuesta correcta: 3

Comentario

Existen dudas respecto a la eficacia del calcio o la vitamina D administradas en monoterapia para el tratamiento de la osteoporosis; señalan que “administrados de forma conjunta parecen poseer un cierto grado de eficacia en la prevención de fracturas no vertebrales, que es más claro en personas deficitarias en ellos, como puede ser el caso de ancianos que viven en residencias. No hay prácticamente evidencia de su eficacia en la prevención de fracturas vertebrales.

Esta pregunta le hemos dado muchas vueltas, consideramos que más que de traumatología el enfoque podría ser más de reumatología.

Mucha suerte a todos y después de corregir, a descansar, que bien os lo habéis ganado

LAS RESPUESTAS TAMBIÉN SE PUBLICARÁN EN LOS DIFERENTES BLOGS DE LOS PROFESIONALES QUE PARTICIPAN EN LA INICIATIVA.

Puedes comentar sobre este tema en el foro

OS PEDIMOS DESDE AQUÍ LA MÁXIMA DIFUSIÓN A LA INICIATIVA PARA QUE LLEGUE AL MÁXIMO DE OPOSITORES MIR PRESENTES Y FUTUROS PARA QUE LES SEA DE AYUDA.

Preguntas MIR 2021: Cirugía Ortopédica y Traumatología.

Respuestas cortesía de Miguel Vázquez (@miguelvazquezdr), Antonio Sánchez (@ansanfer_dr) y Cristina Ojeda-Thies (@ojedathies).
Estas respuestas no son las oficiales ( las actualizaremos con las mismas en cuanto salgan) sólo reflejan la opinión de aquellos voluntarios que han elegido participar en la Iniciativa MIR 2.0 de respuesta al examen MIR de marzo de 2021. (La numeración se corresponde con la Versión 0 del examen)

 

**EDITO: Edito el post una vez el ministerio ha sacado las respuestas que dan por buenas. Distinguiré en aquellas preguntas la respuesta que dimos al revisar el examen y las que el ministerio ha dado por buenas. 

Post publicado en casimedicos.com: 

https://www.casimedicos.com/respuestas-comentadas-a-las-preguntas-de-traumatologia-del-examen-mir-2021/

PREGUNTA 19 

Pregunta asociada a la imagen 19.

Niño de 8 años y medio que acude a urgencias por dolor inguinal izquierdo y claudicación de la marcha tras accidente deportivo jugando al fútbol (impacto con un contrario con posterior caída sobre cadera izquierda). Los padres relatan algún episodio previo similar que mejoró con reposo y tratamiento médico, sin llegar a haber acudido a urgencias. Es un apasionado deportista que juega al fútbol y otros deportes, aunque como dicen los padres últimamente ha debido parar en más de una ocasión. ¿A la vista de la imagen radiográfica cuál sería su diagnóstico clínico?:

Perthes MIR 2021
  1. Fractura de cadera como consecuencia de la caída sufrida.
  2. Epifisiolisis traumática de cadera izquierda.
  3. Artritis séptica de cadera izquierda.
  4. Enfermedad de Perthes.

Respuesta correcta: 4. Enfermedad de Perthes.

Razonamiento: El caso clínico que describe es sugestivo de un cuadro de aparición insidiosa, y la radiología confirma la enfermedad de Perthes de la cadera izquierda. Todas las demás opciones son de clínica más aguda. La epifisiolisis de cadera es más propia de niños mayores que el descrito en el caso (prepubertad). La fractura de cadera queda descartada por la descripción del caso clínico y la radiología. Y la artritis séptica cursaría a menudo con clínica sistémica y hallazgos analíticos (no dados aquí), además de tener una radiología normal en fases precoces. 

Fuente bibliográfica: Marco F (Ed.) Traumatología y Ortopedia para el grado en Medicina. Elsevier, Madrid. 2015. ISBN 9788480866774 : 

Págs 280 – 2018: “La enfermedad de Perthes suele manifestarse de un modo insidioso y progresivo, sin que se asocie ninguna sintomatología general La presentación clínica más habitual es una discreta cojera no dolorosa, que a menudo pasa inadvertida. En otros casos el cuadro se inicia con dolor, bien como una cadera dolorosa, bien como una cojera

dolorosa.… El dolor puede incrementarse o presentarse con las actividades físicas y suele ceder con el reposo. Debido a la baja intensidad y especificidad de la sintomatología, los padres no suelen solicitar atención médica hasta varias semanas después de iniciarse el cuadro clínico.

Los signos radiológicos más precoces son el ensanchamiento del espacio articular de la cadera, la disminución del tamaño, el aumento de la densidad del núcleo de osificación de la cabeza femoral y el ensanchamiento del cuello femoral.”

PREGUNTA 20

Pregunta asociada a la imagen 20.

¿Cuál es el tratamiento de elección para una paciente de 96 años que presenta la fractura de la imagen?:

Preguntas traumatología MIR 2021
  1. Clavo trocantérico en mesa ortopédica.
  2. Prótesis parcial bipolar de cadera.
  3. Osteosíntesis con tornillos canulados.
  4. No cirugía, vida cama-sillón precoz.

Respuesta: 1. Clavo trocantérico en mesa ortopédica

Razonamiento: La radiografía nos muestra una fractura trocantérea (pertrocantérea / intertrocantérea), el tratamiento más comúnmente empleado para este tipo de fractura es el clavo trocantérico. Para algunas fracturas trocantéreas (más sencillas que la de la radiografía) se podría plantear un tornillo-placa deslizante o DHS, pero no se ofrece esta opción en la pregunta. La prótesis parcial bipolar de cadera es el tratamiento más empleado en las fracturas intracapsulares (transcervicales / subcapitales) desplazadas, y la osteosíntesis con tornillos canulados en las fracturas intracapsulares no desplazadas, sobre todo en pacientes más jóvenes. El manejo cnservador con vida cama-sillón se reserva excepcionalmente para pacientes con fracturas intracapsulares con mucha comorbilidad y por tanto un elevadísimo riesgo quirúrgico / anestésico, con una función previa a la fractura limitada (que no caminan); ningún dato del enunciado nos apunta hacia esta opción.

Fuente bibliográfica: Marco F (Ed.) Traumatología y Ortopedia para el grado en Medicina. Elsevier, Madrid. 2015. ISBN 9788480866774 : 

Págs 309 – 310: “Son fracturas extracapsulares y entre ellas se encuentran aquellas cuyo trazo o trazos se localizan entre ambos trocánteres (intertrocantéreas), proximales a esta zona pero con prolongación distal (pertrocantéreas) o distales a nivel del trocánter menor (subtrocantéreas).

…Son fracturas habitualmente con varios trazos fracturarios y generalmente inestables, ya que únicamente serán estables cuando la cortical medial o calcar mantenga un buen

contacto y apoyo de los fragmentos y si estos se limitan a dos, es decir, cuando se trata de fracturas simples.

…Se considera que todas estas fracturas presentan indicación quirúrgica, incluidas las fracturas consideradas estables en la radiografía, dada la alta posibilidad de desplazamiento

secundario. Existen dos tipos de dispositivos estándares utilizados en la osteosíntesis de las fracturas trocantéreas: por un lado el tornillo-placa deslizante de cadera (fig. 28.5), de localización extramedular, y, por otro, el clavo trocantéreo (fig. 28.6), que consiste en un clavo endomedular asociado a un tomillo cervical.

En general, el tornillo-placa deslizante está más indicado en fracturas estables, por ser una técnica quirúrgica simple y con buenos resultados. En fracturas inestables, los dispositivos intramedulares, como el clavo trocantéreo, parecen estar más indicados, en especial en fracturas con conminución posteromedial, extensión subtrocantérea o trazo invertido.”

PREGUNTA 21

Pregunta asociada a la imagen 21.

Paciente de 44 años con lesión (16x8x12 cm) de crecimiento rápido que interesa las partes blandas del tercio proximal del muslo derecho. Refiere que ha empezado a notar la masa, que tiene una consistencia dura y está adherida a planos profundos, en los últimos 6 meses. Previamente hacía deporte habitualmente, llegando a haber terminado alguna maratón. Se observa a la exploración circulación colateral, pero no se palpan adenopatías inguinales derechas. Tras la biopsia se confirma que se trata de un sarcoma pleomórfico indiferenciado de alto grado, que en los estudios de imagen se encuentra proximal al trocánter menor. La TC de extensión es negativa. ¿Cuál sería su actitud terapéutica?:

sarcoma pleomorfico MIR 2021
  1. Desarticulación de la cadera.
  2. Tratamiento con ifosfamida y tamoxifeno durante 6 ciclos previos a la cirugía de resección para reducir el tamaño de la lesión.
  3. Cirugía marginal extirpando la pseudocápsula que suelen formar estos tumores como respuesta a su rápido crecimiento.
  4. Hemipelvectomía derecha modificada.

Respuesta correcta del ministerio: La 4. 

 

Nuestra respuesta inicial: La 2. Tratamiento con ifosfamida y tamoxifeno durante 6 ciclos previos a la cirugía de resección para reducir el tamaño de la lesión. 

 

Nuestro razonamiento inicial: 

Razonamiento: En los márgenes de resección, tenemos: intralesional → marginal (opción 3) → Ampliada → Radical (opciones 1 y 4). Se trata de un tumor de alto grado, en el que está indicada la resección ampliada que incluye márgenes libres del tumor. La cirugía radical de amputación no ha demostrado mejorar la supervivencia reservándose para casos seleccionados (invasión de estructuras neurovasculares, fractura patológica…). Por lo tanto, descartamos tanto la opción 1 como 4; la opción 3 también queda descartada, porque es un margen menor que la ampliada; por eliminación nos quedamos con la 2.

Fuente bibliográfica: Marco F (Ed.) Traumatología y Ortopedia para el grado en Medicina. Elsevier, Madrid. 2015. ISBN 9788480866774 :

Pág 181: 

Cirugía de resección

Teniendo en cuenta los márgenes a cuyo nivel se realice la extirpación del tumor, hay que considerar varios tipos de resección: intralesional o intratumoral, marginal, ampliada y radical.

  • En la resección intralesional se hace el abordaje directo del tumor y su extirpación atravesando el mismo o, si es cavitario, vaciando su contenido mediante legrado y raspado de la pared. Esta tipo de resección solo está indicada en lesiones benignas que están limitadas por una cápsula o que son cavitarias (encondromas, lesiones quísticas, tumores de células gigantes y algunos condroblastomas).
  • La resección marginal se hace a nivel de la periferia o en las proximidades del tumor y por fuera de la cápsula, justamente en la zona reactiva que la rodea. Estaría indicada en tumores benignos activos y agresivos (tumores agresivos de células gigantes, condroblastomas agresivos, osteocondromas, etc.).
  • La resección con márgenes amplios, o resección ampliada, se hace lejos de la periferia del tumor y de su zona reactiva y siempre en tejido sano a una distancia mínima de 1 cm de la lesión (fig. e l 7 .10). En realidad se trata de una resección intracompartimental y sobre todo se utiliza en tumores malignos, tanto de bajo como de alto grado, intracompartimentales. En la actualidad, la cirugía asistida por «navegación» está empezando a utilizarse con el objetivo de conseguir una mejor selección de los márgenes de resección.
  • La resección de tipo radical se realiza por fuera de los límites del compartimento, lo que implica la extirpación total del mismo junto con el tumor. Esta resección estaría indicada básicamente en el caso de tumores de alta malignidad y gran extensión local. Las amputaciones y desarticulaciones de los miembros son formas de resección radical pero muy agresivas y mutilantes, que obligan al sacrificio del miembro, por lo que están indicadas en los casos en que no lo está una cirugía de «salvación», circunstancia que se da en los siguientes casos: tumores con gran invasión de partes blandas, cuando existe infiltración de los ejes neurovasculares, en infecciones y en secuelas de la radioterapia; también se valora en los casos que se complican con una fractura patológica. Los progresos obtenidos con la cirugía conservadora han limitado mucho en la actualidad las indicaciones de la cirugía mutilante.

Pág 190: TUMORES MALIGNOS (SARCOMAS DE PARTES BLANDAS)

En principio, el tratamiento tiene como objetivo la resección de la lesión con los márgenes de seguridad adecuados y si es posible la conservación del miembro afectado (cirugía

de salvación del miembro). Los márgenes de resección en estos tumores deben ser amplios o radicales. En el caso de que esta resección no haya sido lo suficientemente amplia

(al menos de 2-3 cm de margen), las tasas de recurrencia local suelen ser muy altas. No obstante, y a pesar de que la resección sea amplia, estas recurrencias pueden llegar

a ser de hasta el 25%. 

Pág 191: Histiocitoma fibroso maligno. Es el más frecuente de los sarcomas de partes blandas (40% de todos ellos). Se observa generalmente en adultos varones por encima de los 50 años, se localiza en tejidos profundos de las extremidades inferiores y sobre todo en el muslo, presentándose como una masa de crecimiento lento poco o nada dolorosa. Existen diferentes tipos histológicos: mixoide, angiomatoide y estoriforme-pleomórfico, siendo este último el más frecuente. Suelen dar metástasis pulmonares y muy pocas veces linfáticas. El tratamiento es la resección ampliada (figs. e l 8 .6 -e l8 .8 ).

Nuestra valoración actual: 

Nos parece que la pregunta pueda ser impugnable. Aquí os damos los argumentos para defender su impugnación. 

ARGUMENTO PARA LA IMPUGNACIÓN:

Pregunta a nuestro parecer IMPUGNABLE. Ninguna opción es 100% correcta.  Las opciones 1 y 4 serían márgenes radicales, la 3 marginal, la 2 parece que ampliada, pero el protocolo de quimioterapia neoadyuvante no es válido. 

En el enunciado de la pregunta no hay ningún dato que sugiera invasión de estructuras neurovasculares o invasión de partes blandas que impida la resección ampliada y que obligue a una cirugía radical, cosa que en cualquier caso habría que confirmar con resonancia magnética. 

Es más, el tratamiento mediante cirugía radical, muy mutilante, no ha demostrado mejorar la supervivencia (hay un porcentaje importante de pacientes con micrometástasis en el momento del diagnóstico, no detectables mediante el estudio de extensión), aparte de limitar la calidad de vida. La pauta más habitual es referir al paciente a un centro especializado en sarcomas, para manejo multidisciplinar, que suele consistir en una combinación de cirugía y radioterapia, a menudo también con quimioterapia neoadyuvante, que tiene varias funciones: busca la necrosis tisular local, la respuesta al tratamiento quimioterápico ayuda en estimar el pronóstico – sobre todo si más adelante se detectan recurrencias o metástasis-, y el tratamiento sistémico puede controlar enfermedad a distancia. 

La quimioterapia neoadyuvante contiene a menudo ifosfamida y doxorrubicina en 3 ciclos, mientras que el tamoxifeno (opción 2) es un agente hormonal empleado más a menudo para el cáncer de mama, y no se suelen emplear 6 ciclos, por lo cual la opción 2 también es nula. 



Bibliografía: 

  1. Marco F (Ed.) Traumatología y Ortopedia para el grado en Medicina. Elsevier, Madrid. 2015. ISBN 9788480866774. Pág 181 
  2. Gilbert NF, Cannon CP, Lin PP, Lewis VO. Soft-tissue sarcoma. J Am Acad Orthop Surg. 2009 Jan;17(1):40-7. doi: 10.5435/00124635-200901000-00006. PMID: 19136426.
  3. Luis ÁM, Aguilar DP, Martín JA. Multidisciplinary management of soft tissue sarcomas. Clin Transl Oncol. 2010 Aug;12(8):543-53. doi: 10.1007/s12094-010-0552-2. PMID: 20709652.
  4. Cable MG, Randall RL. Extremity Soft Tissue Sarcoma: Tailoring Resection to Histologic Subtype. Surg Oncol Clin N Am. 2016 Oct;25(4):677-95. doi: 10.1016/j.soc.2016.05.014. Epub 2016 Aug 5. PMID: 27591492.

 

  1. Scoggins CR, Pisters PW. Diagnosis and management of soft tissue sarcomas. Adv Surg. 2008;42:219-28. doi: 10.1016/j.yasu.2008.04.002. PMID: 18953820.
  2. Grobmyer SR, Maki RG, Demetri GD, Mazumdar M, Riedel E, Brennan MF, Singer S. Neo-adjuvant chemotherapy for primary high-grade extremity soft tissue sarcoma. Ann Oncol. 2004 Nov;15(11):1667-72. doi: 10.1093/annonc/mdh431. PMID: 15520069.

 

Bibliografía con texto de interés. 

  1. Marco F (Ed.) Traumatología y Ortopedia para el grado en Medicina. Elsevier, Madrid. 2015. ISBN 9788480866774. Pág 181 

La cirugía de resección tumoral tiene 4 tipos de márgenes: 

  1. Intralesional: consiste en el abordaje directo del tumor y su extirpación atravesando el mismo o, si es cavitario, vaciando su contenido mediante legrado y raspado de la pared. Esta tipo de resección solo está indicada en lesiones benignas que están limitadas por una cápsula o que son cavitarias (encondromas, lesiones quísticas, tumores de células gigantes y algunos condroblastomas).
  2. Marginal: a nivel de la periferia o en las proximidades del tumor y por fuera de la cápsula, justamente en la zona reactiva que la rodea. Estaría indicada en tumores benignos activos y agresivos (tumores agresivos de células gigantes, condroblastomas agresivos, osteocondromas, etc.).
  3. Ampliada: Incluye mínimo 1 cm de tumor sano alrededor del tumor. En realidad se trata de una resección intracompartimental y sobre todo se utiliza en tumores malignos, tanto de bajo como de alto grado, intracompartimentales. 
  4. Radical: se realiza por fuera de los límites del compartimento, lo que implica la extirpación total del mismo junto con el tumor. Esta resección estaría indicada básicamente en el caso de tumores de alta malignidad y gran extensión local. Las amputaciones y desarticulaciones de los miembros son formas de resección radical pero muy agresivas y mutilantes, que obligan al sacrificio del miembro, por lo que están indicadas en los casos en que no lo está una cirugía de «salvación», circunstancia que se da en los siguientes casos: tumores con gran invasión de partes blandas, cuando existe infiltración de los ejes neurovasculares, en infecciones y en secuelas de la radioterapia; también se valora en los casos que se complican con una fractura patológica. Los progresos obtenidos con la cirugía conservadora han limitado mucho en la actualidad las indicaciones de la cirugía mutilante.



  1. Gilbert NF, Cannon CP, Lin PP, Lewis VO. Soft-tissue sarcoma. J Am Acad Orthop Surg. 2009 Jan;17(1):40-7. doi: 10.5435/00124635-200901000-00006. PMID: 19136426.

The mainstay of local treatment is excision with wide surgical margins, whenever possible. Positive margins are a risk factor for local recurrence; however, local recurrence has not been shown to affect overall survival in patients with soft-tissue sarcoma.28,29 Therefore, acceptance of close margins around vital structures is acceptable to preserve a functional limb. Limb salvage is possible in most cases, with amputation reserved for the patient in whom an adequate margin cannot be obtained with limb-sparing surgery. 

Preoperative administration of chemotherapy may facilitate limb salvage by causing necrosis of the primary tumor, thereby allowing easier tumor resection. At our center, chemotherapy is usually given as a neoadjuvant treatment for highrisk tumors (American Joint Commission on Cancer stage III) or for metastatic disease (stage IV). Patients with high-grade, large, deep lesions have approximately a 50% chance of developing metastatic disease; thus, chemotherapy frequently is employed for these patients. Agents commonly used in the treatment of soft-tissue sarcoma include doxorubicin and ifosfamide. 

 

  1. Luis ÁM, Aguilar DP, Martín JA. Multidisciplinary management of soft tissue sarcomas. Clin Transl Oncol. 2010 Aug;12(8):543-53. doi: 10.1007/s12094-010-0552-2. PMID: 20709652.

Amputation has been a mainstay in managing extremity STS, being used in >40% of cases [19, 20] until the early 1970s, when the hypothesis was raised that radiotherapy in combination with surgery could achieve an equivalent result [5]. Conclusions of several randomised studies confi rmed the appropriateness of limb-sparing surgery in combination with radiotherapy for a significant proportion of patients with high-grade extremity sarcomas, thus establishing a new era in their management [21]. Although it was originally thought that large high-grade sarcomas might have a lesser risk of metastasis if treated by amputation, Williard et al. reported no survival advantage for this group when treated by amputation compared with limbsparing surgery [22]… However, there remains a group of patients that is better served by an amputation rather than by a limb-sparing procedure. Consideration of amputation should be made if one or more of the following tumour characteristics occur [14] and resection of the tumour is expected to render the limb nonfunctional [3]: 

  1. Extensive soft tissue mass and/or skin involvement 
  2. Involvement of a major artery or nerve 
  3. Extensive bony involvement necessitating whole bone resection 
  4. Failure of preoperative chemotherapy or radiation therapy 
  5. Tumour recurrence after previous adjuvant radiation

 

  1. Cable MG, Randall RL. Extremity Soft Tissue Sarcoma: Tailoring Resection to Histologic Subtype. Surg Oncol Clin N Am. 2016 Oct;25(4):677-95. doi: 10.1016/j.soc.2016.05.014. Epub 2016 Aug 5. PMID: 27591492.

 

  1. Scoggins CR, Pisters PW. Diagnosis and management of soft tissue sarcomas. Adv Surg. 2008;42:219-28. doi: 10.1016/j.yasu.2008.04.002. PMID: 18953820.

For extremity STS, the goal of resection should be a limb‐sparing, function‐preserving oncologic resection with adequate margins… As a result, current rates of amputations for patients with STS are approximately <5% for those with primary tumors or from 9% to 14% for recurrent disease, with such procedures reserved for cases in which resection or reresection with adequate margins cannot be performed without sacrificing the functional outcome of the limb.53, 54

More recently, Gronchi et al completed a phase 3 randomized controlled trial in which high‐risk patients with high‐grade, deep, >5‐cm truncal or extremity tumors were randomized to receive standard neoadjuvant chemotherapy (anthracycline and ifosfamide) or histology‐tailored neoadjuvant chemotherapy for 5 specific sarcoma histiotypes, including UPS, myxoid liposarcoma (MLS), synovial sarcoma, malignant peripheral nerve sheath tumor, and leiomyosarcoma. With a median follow‐up duration of 12.3 months, the projected DFS rate at 46 months was 62% in the standard chemotherapy group and 38% in the histiotype‐tailored chemotherapy group (P = .004).77 After a longer follow‐up, the histiotype‐tailored chemotherapy group had better DFS than initially detected, suggesting some effect of the histiotype‐tailored chemotherapy. That trial confirms the value of neoadjuvant chemotherapy in patients with high‐risk truncal or extremity STS and further highlights the possibility of histology‐specific recruitment strategies in future randomized trials. There have also been other small retrospective series that have attempted to identify a cohort of patients with extremity STS who might benefit from neoadjuvant chemotherapy.78, 79 These studies suggest that there may be a high‐risk group of patients, such as those with high‐grade tumors measuring >10 cm, for whom neoadjuvant chemotherapy can be considered. Finally, the local impact of preoperative treatments should not be overlooked. In other words, although the primary aim of neoadjuvant chemotherapy in operable patients is systemic, a local benefit is likely to occur at least in a proportion of patients (Fig. 4). The preoperative combination of chemotherapy with radiation was shown to be feasible and to offset the adverse impact of positive surgical margins.80, 81 Function preservation may also be part of this benefit.

 

  1. Grobmyer SR, Maki RG, Demetri GD, Mazumdar M, Riedel E, Brennan MF, Singer S. Neo-adjuvant chemotherapy for primary high-grade extremity soft tissue sarcoma. Ann Oncol. 2004 Nov;15(11):1667-72. doi: 10.1093/annonc/mdh431. PMID: 15520069.

 

PREGUNTA 108

En una luxación posterior de la cadera, señale la respuesta INCORRECTA:

  1. El miembro inferior se encuentra en flexión, rotación externa y abducción.
  2. Hay que realizar una reducción de urgencia.
  3. Una de sus complicaciones posibles es la necrosis de la cabeza femoral.
  4. Una vez reducida, son infrecuentes las recidivas.

Respuesta correcta: 1. El miembro inferior se encuentra en flexión, rotación externa y abducción. 

Razonamiento: En la luxación posterior de cadera, el miembro inferior se encuentra en flexión, rotación INTERNA y ADDUCCION (posición de bañista sorprendido). Las demás respuestas son verdaderas. 

Fuente bibliográfica: Marco F (Ed.) Traumatología y Ortopedia para el grado en Medicina. Elsevier, Madrid. 2015. ISBN 9788480866774 : 

Pág 265: “En la luxación posterosuperior o ilíaca, la más frecuente, el miembro se presenta acortado, en flexión de 20°, aducción y rotación interna, posición conocida como de la «bañista sorprendida» (v. fig. e24.11). En la luxación posteroinferior, la cabeza se coloca sobre el isquion, y el miembro inferior, en flexión marcada, aducción y rotación interna.

PREGUNTA 106

Una paciente de 65 años sufrió hace tres semanas una caída con fractura-luxación anterior de hombro que fue reducida en urgencias. Le fue colocado un cabestrillo y se recomendó su retirada a las 3 semanas. Al retirar la inmovilización se observa una zona de disestesia circunscrita a la región lateral del hombro. La paciente puede realizar abducción, pero solo alcanza 15º. Dispone de un estudio de resonancia magnética en el que el manguito de los rotadores está indemne. ¿Cuál es el diagnóstico de sospecha más probable?

  1. Tendinopatía del redondo mayor.
  2. Capsulitis adhesiva.
  3. Neuropatía del nervio supraescapular.
  4. Lesión del nervio axilar

Respuesta correcta: 4. Lesión del nervio axilar. 

Razonamiento: La disestesia de la cara lateral de la región deltoidea del hombro (zona inervada por el nervio axilar) y la parálisis del deltoides (músculo primario para la abducción más allá de 15º). El manguito de rotadores está indemne en la resonancia magnética, descartando la opción 1 (nos mostraría la tendinopatía del redondo mayor). La capsulitis adhesiva cursa con dolor y rigidez del hombro, sobre todo a la rotación externa tanto activa como pasiva. El nervio supraescapular inercia el músculo supraespinoso, que contribuye a los primeros 15º de abducción del hombro (conservados en el enunciado) y no inerva la piel de la zona reflejada.

PREGUNTA 107

En la patología de la mano, ¿cuál de las siguientes afirmaciones sobre la enfermedad de Dupuytren es INCORRECTA?:

  1. Afecta con mayor frecuencia a los dedos anular y meñique.
  2. El tratamiento conservador con fisioterapia es poco eficaz.
  3. Es un engrosamiento y retracción de la aponeurosis palmar.
  4. Es más frecuente en trabajadores manuales.

Respuesta correcta del ministerio: 4

ARGUMENTO PARA LA IMPUGNACIÓN:

Pregunta a nuestro parecer IMPUGNABLE, ya que todas las opciones dadas son correctas. Consideramos que con la bibliografía actual puede defenderse que el enunciado 4 también es correcto.

Razonamiento: la Enfermedad de Dupuytren afecta por orden de frecuencia los dedos 4º -> 5º -> 3º -> 2º o 1º. Es un una metaplasia de las células de la aponeurosis palmar que da lugar al engrosamiento y la retracción de la misma. Se ha descrito como factor de riesgo, entre otros, los traumatismos repetidos. La asociación con  la ocupación es un tema de bastante debate – si se acepta que es más frecuente en trabajadores expuestos a vibraciones, y varios trabajos recientes sí que la reconocen como enfermedad ocupacional, con un odds ratio de al menos 2. La fisioterapia es poco eficaz; aunque el tratamiento con ortesis, infiltraciones y ondas de choque puede retrasar la progresión de la enfermedad en fases precoces, pero no han demostrado la eficacia de la cirugía.

 

Bibliografía: 



  1. Marco F (Ed.) Traumatología y Ortopedia para el grado en Medicina. Elsevier, Madrid. 2015. ISBN 9788480866774 : Pág 254:   
  2. Green’s Cirugía de la mano. Capítulo 5  Contractura de Dupuytren Autor: D.A. McGrouther Pg 161
  3. Lurati AR. Dupuytren’s Contracture. Workplace Health Saf. 2017 Mar;65(3):96-99. doi: 10.1177/2165079916680215. Epub 2017 Jan 9. PMID: 28068478.
  4. Descatha A, Jauffret P, Chastang JF, Roquelaure Y, Leclerc A. Should we consider Dupuytren’s contracture as work-related? A review and meta-analysis of an old debate. BMC Musculoskelet Disord. 2011 May 16;12:96. doi: 10.1186/1471-2474-12-96. PMID: 21575231; PMCID: PMC3123614.
  5. Liss GM, Stock SR. Can Dupuytren’s contracture be work-related?: review of the evidence. Am J Ind Med. 1996 May;29(5):521-32. doi: 10.1002/(SICI)1097-0274. PMID: 8732927.
  6. Alser OH, Kuo RYL, Furniss D. Nongenetic Factors Associated with Dupuytren’s Disease: A Systematic Review. Plast Reconstr Surg. 2020 Oct;146(4):799-807. doi: 10.1097/PRS.0000000000007146. PMID: 32970002.ç
  7. Descatha A, Bodin J, Ha C, Goubault P, Lebreton M, Chastang JF, Imbernon E, Leclerc A, Goldberg M, Roquelaure Y. Heavy manual work, exposure to vibration and Dupuytren’s disease? Results of a surveillance program for musculoskeletal disorders. Occup Environ Med. 2012 Apr;69(4):296-9. doi: 10.1136/oemed-2011-100319. Epub 2012 Jan 2. PMID: 22213840; PMCID: PMC3815440.

 

Bibliografía con texto de interés. 

 

  1. Marco F (Ed.) Traumatología y Ortopedia para el grado en Medicina. Elsevier, Madrid. 2015. ISBN 9788480866774 : 

Pág 254: “ENFERMEDAD DE DUPUYTREN: 

Esta enfermedad consiste en un engrosamiento nodular con acortamiento de la aponeurosis o fascia palmar. Es idiopática pero se ha descrito un comportamiento genético dominante, que se da especialmente en personas del norte de Europa. Afecta sobre todo a varones mayores de 50 años y se han asociado como factores de riesgo la epilepsia, la diabetes, la enfermedad pulmonar crónica, el alcoholismo, el tabaquismo y los microtraumatismos de repetición. 

… El dedo más frecuentemente afectado es el cuarto, seguido, por orden de frecuencia, del quinto, del tercero, del segundo y del primero.

… El empleo de férulas no es curativo, aunque puede retrasar la progresión de las contracturas. El tratamiento definitivo es quirúrgico… “

  

  1. Green’s Cirugía de la mano. Capítulo 5  Contractura de Dupuytren Autor: D.A. McGrouther Pg 161

El mismo Dupuytren relacionó la contractura con trabajos pesados en un paciente, un chófer, y con un traumatismo en otro, un comerciante de vinos, que había trabajado hacía muchos años a un ritmo muy fuerte levantando barriles. Ninguno de estos dos tipos de casos, en que se presenta el uso derivado de la ocupación, o el traumatismo como causas, han sido sometidos a un escrutinio científico concienzudo

 

  1. Lurati AR. Dupuytren’s Contracture. Workplace Health Saf. 2017 Mar;65(3):96-99. doi: 10.1177/2165079916680215. Epub 2017 Jan 9. PMID: 28068478.

 

  1. Descatha A, Jauffret P, Chastang JF, Roquelaure Y, Leclerc A. Should we consider Dupuytren’s contracture as work-related? A review and meta-analysis of an old debate. BMC Musculoskelet Disord. 2011 May 16;12:96. doi: 10.1186/1471-2474-12-96. PMID: 21575231; PMCID: PMC3123614.

The meta-OR for manual work was 2.02[1.57;2.60] (HQMC studies only: 2.01[1.51;2.66]), and the meta-OR for vibration exposure was 2.88 [1.36;6.07] (HQMC studies only: 2.14[1.59;2.88]).

 

  1. Liss GM, Stock SR. Can Dupuytren’s contracture be work-related?: review of the evidence. Am J Ind Med. 1996 May;29(5):521-32. doi: 10.1002/(SICI)1097-0274. PMID: 8732927.

Bennett [1982: Br J Ind Med 39:98-100] found the prevalence of DC at a British PVC bagging and packing plant in which workers were exposed to repetitive manual work to be 5.5 times that at a local plant without packing, and twice the expected prevalence in a U.K. working population previously studied by Early [1962: J Bone Joint Surg 44B:602-613]. DC was observed more frequently among vibration white finger claimants than controls by Thomas and Clarke [1992: J Soc Occup Med 42:155-158] (OR, 2.1; 95% CI, 1.1-3-9), and more frequently among vibration-exposed workers than controls by Bovenzi et al. [1994: Occup Environ Med 51:603-611] (OR, 2.6 95% CI, 1.2-5.5). Cocco et al [1987: Med Lav 78:386-392] found that a history of vibration exposure occurred more frequently among cases of DC than among controls (OR, 2.3; 95% CI, 1.5-4.4). The latter two studies presented some evidence of a dose-response relationship. There is good support for an association between vibration exposure and DC.

  1. Alser OH, Kuo RYL, Furniss D. Nongenetic Factors Associated with Dupuytren’s Disease: A Systematic Review. Plast Reconstr Surg. 2020 Oct;146(4):799-807. doi: 10.1097/PRS.0000000000007146. PMID: 32970002.ç

There was strong evidence for the association between Dupuytren’s disease and advanced age, male sex, family history of Dupuytren’s disease, and diabetes mellitus. Furthermore, heavy alcohol drinking, cigarette smoking, and manual work exposure showed a significant dose-response relationship.

 

  1. Descatha A, Bodin J, Ha C, Goubault P, Lebreton M, Chastang JF, Imbernon E, Leclerc A, Goldberg M, Roquelaure Y. Heavy manual work, exposure to vibration and Dupuytren’s disease? Results of a surveillance program for musculoskeletal disorders. Occup Environ Med. 2012 Apr;69(4):296-9. doi: 10.1136/oemed-2011-100319. Epub 2012 Jan 2. PMID: 22213840; PMCID: PMC3815440.

Heavy manual work without vibration exposure was significantly associated with the condition (adjusted OR (aOR) 3.9; 95% CI 1.3 to 11.5) adjusted on age and diabetes), as was use of vibrating tools (aOR 5.1; 2.1 to 12.2). These associations remained significant among subjects with >10 years in the same job, with increases in aOR of 6.1 (1.5 to 25.0) and 10.7 (3.4 to 34.6), respectively.

PREGUNTA 109

Un niño sufre, en un accidente deportivo, una epifisiolisis distal de tibia. Al informar a los padres sobre el pronóstico, ¿cuál de los siguientes factores NO influye en el riesgo de deformidad futura?:

  1. Edad y género del niño.
  2. Fractura intraarticular.
  3. Tipo de Salter y Harris.
  4. Actividad deportiva.

Respuesta correcta: 4. Actividad deportiva

Razonamiento: La gravedad de las epifisiolisis depende fundamentalmente del potencial de alterar el crecimiento futuro del hueso afectado. Eso depende de la edad y del género del niño (peor en niños más inmaduros esqueléticamente, y los niños tienen una pubertad más tardía que las niñas. El riesgo de que se altere el crecimiento restante de la fisis depende del tipo de epifisiolisis (Salter y Harris), las que tienen afectación intraarticular (Salter y Harris III y IV) tienen un elevado riesgo de afectar el crecimiento. La actividad deportiva no influye. 

Fuente bibliográfica: 

Marco F (Ed.) Traumatología y Ortopedia para el grado en Medicina. Elsevier, Madrid. 2015. ISBN 9788480866774 : 

Págs 78-79: 

“Grado III: menos frecuente que la anterior (10% aproximadamente), pero de mayor repercusión sobre el crecimiento. El trazo afecta a una parte del cartílago de conjunción, para hacerse después perpendicular al mismo y hendir la epífisis respetando la metáfisis. El trazo de fractura va a lesionar en un punto la capa germinal fisaria y, además, la superficie articular, con lo que se convertirá en una fractura intraarticular.

Grado IV: menos frecuente que las anteriores (6 %); equivale a una fractura por separación epifisometafisaria del adulto. Se produce por un mecanismo de compresión-flexión que separa por un trazo vertical un fragmento epifisometafisario, con lesión del cartílago

de conjunción en todas sus capas y de la superficie articular. Exige una reducción anatómica por su afectación doble (articular y fisaria). Las detenciones del crecimiento van a ser frecuentes, con la aparición de dismetrías y/o deformidades angulares, si se produce un

bloqueo parcial del crecimiento de la fisis.

… Las repercusiones sobre el crecimiento longitudinal serán mayores cuanto más pequeño sea el niño, ya que el crecimiento quedará anulado durante más tiempo. Las fisis en torno a la rodilla y las alejadas del codo son las más activas y, en consecuencia, su lesión originará un mayor acortamiento, que tendrá mayor importancia clínica en los miembros inferiores por las dismetrías que produce”

PREGUNTA 110

Paciente de 45 años que presenta un primer episodio de lumbalgia de 3 semanas de evolución que le impide realizar su vida normal. No refiere traumatismo ni otra patología subyacente. En la exploración física no hay déficit neurológico. ¿Qué prueba de imagen estaría indicada?:

  1. Al ser el primer episodio de lumbalgia, únicamente una radiografía simple de columna lumbar.
  2. Resonancia magnética, que aporta más información sobre partes blandas y posibles herniaciones.
  3. TC, para valorar mejor la estructura ósea y las posibles fracturas.
  4. No hay indicación de prueba de imagen.

Respuesta correcta:  4. No hay indicación de prueba de imagen.

Razonamiento: Un primer episodio de lumbalgia aguda (menos de 6 semanas de duración), no requiere la realización de pruebas de imagen en ausencia de “red flags” que puedan indicar un proceso subyacente, como una fractura, una infección o un tumor maligno.

Fuente bibliográfica: Delgado Martínez (Ed). Cirugía Ortopédica y Traumatología. Panamericana, Madrid. 2018. ISBN: 9788491103479, capítulo 78:

Estudio por imagen:

  1. Radiografías:
    1. AP-L (deben ser el primer estudio):
      1. No es necesario hacerlas siempre. Sólo tras lumbalgia mecánica de 6 semanas de evolución, salvo que existan signos de alarma (“banderas rojas” (tabla 3)) indicativos de malignidad o infección.

PREGUNTA 111

Un paciente de 40 años comienza a sentir molestias en la nalga derecha tras un viaje prolongado en coche. Desde ese momento no tolera la sedestación prolongada por reaparición del dolor. Ocasionalmente siente entumecimiento en el miembro inferior derecho que desaparece al levantarse. Dispone de estudio de resonancia magnética y radiografía de cadera en los que no se informan anomalías. El examen de la sensibilidad y los reflejos osteotendinosos no muestra alteraciones. Las maniobras de rotación externa y la abducción resistidas, con la cadera en 90 grados de flexión, reproducen el dolor. El diagnóstico más probable es:

  1. Síndrome de choque fémoro-acetabular.
  2. Síndrome del tensor de la fascia lata.
  3. Radiculopatía S1.
  4. Síndrome del piramidal.

Respuesta correcta: 4. Síndrome del piramidal.

Razonamiento: El síndrome del piramidal se produce debido a la compresión o pinzamiento del nervio ciático por hipertrofia o contractura del músculo piriforme.

Los síntomas característicos del síndrome del piramidal incluyen un dolor glúteo irradiando la cara posterior del muslo que puede acompañarse de parestesias. Como desencadenante frecuente el paciente puede referir un largo periodo de sedestación sobre una superficie dura previo al dolor. 

El caso clínico expone la maniobra de PACE en la que el paciente resiste al examinador la rotación externa y la abducción de la cadera a 90º de flexión. Se considera positivo si reproduce los síntomas en contexto de síndrome del piramidal.

Fuente bibliográfica: Delgado Martínez (Ed). Cirugía Ortopédica y Traumatología. Panamericana, Madrid. 2018. ISBN: 9788491103479, capítulo 53:

  • Exploración: Aumenta el dolor a la rotación externa contra resistencia de cadera o en rotación interna pasiva con la cadera en extensión (tensan el músculo).

PREGUNTA 180

Paciente de 25 años de edad, que sufre un accidente de moto un viernes por la noche. Es trasladado a urgencias y diagnosticado de traumatismo abdominal (eco-fast negativa), un traumatismo craneal leve (Glasgow = 14) y una fractura intracapsular desplazada de cadera derecha. Hemodinámicamente está estable. ¿Cuál sería el tratamiento de elección?:

  1. Reducción, abierta si se precisa, y osteosíntesis de la fractura en las primeras 24-36 horas.
  2. Artroplastia total de superficialización de la cadera el lunes de forma programada.
  3. Esperar a que mejore del traumatismo craneal y programar la semana siguiente una cirugía reglada consistente en la reducción y osteosíntesis de la fractura.
  4. Ante el riesgo de no consolidación de estos tipos de fractura le implantaría de urgencias una hemiartroplastia bipolar de cadera.

Respuesta correcta: 1

Razonamiento: El manejo de la fractura intracapsular desplazada de cadera en un paciente joven es quirúrgico y debe hacerse precozmente para reducir el riesgo de necrosis avascular de la cadera. 

No nos están hablando de riesgo vital como para aplazar un acto quirúrgico necesario o como para plantearnos un fijador externo siguiendo una política de contención de daño por lo que descartamos la opción 3. Se trata de un paciente de 25 años de modo que tenemos que perseguir la reducción y fijación evitando las opciones de reemplazo (descartamos opción 2 y 4). 

LAS RESPUESTAS TAMBIÉN SE PUBLICARÁN EN LOS DIFERENTES BLOGS DE LOS PROFESIONALES QUE PARTICIPAN EN LA INICIATIVA.

Puedes comentar sobre este tema en el foro

OS PEDIMOS DESDE AQUÍ LA MÁXIMA DIFUSIÓN A LA INICIATIVA PARA QUE LLEGUE AL MÁXIMO DE OPOSITORES MIR PRESENTES Y FUTUROS PARA QUE LES SEA DE AYUDA.

2015 Hemipelvectomía + injerto de peroné

El dibujo que subo hoy fue un encargo que recibí en 2015 durante mi rotatorio en el Servicio de Cirugía Ortopédica y Traumatología del Hospital Sant Joan de Déu de Barcelona.

Me pidieron ilustrar un caso para el artículo que hablaría de ello en diario medico.

La paciente presentaba sarcoma de Ewing de grandes dimensiones en la cresta ilíaca derecha, precisaba una cirugía amplia que obligaba a reconstruir el anillo pélvico. Tras la extirpación se consiguió la reconstrucción con injerto vascularizado de peroné de la pierna izquierda.

Se informó de buena evolución de la paciente.

El primer planteamiento para este dibujo fue plantearlo como tres dibujos para explicar la cirugía en tres puntos.

Se extirpa el tumor de la pala ilíaca derecha mediante una osteotomía y desarticulando la articulación sacroilíaca.
Se extrae injerto de peroné con pedículo vascular de la propia paciente. El injerto se prepara en la mesa de quirófano para luego poderlo anastomosar a vasos sanguíneos de la zona receptora.

Luego me solicitaron cambiarlo y modificarlo para que se vieran los tres pasos en una sola ilustración.

¿Os ha gustado?

Preguntas MIR 2019: Cirugía Ortopédica y Traumatología

La imagen puede contener: texto

Otro año más os traigo comentadas las preguntas del bloque de Cirugía Ortopédica y Traumatología del examen de acceso a la formación MIR de este año.

Agradecimientos a todo el grupo de MIR 2.0 y la enhorabuena porque creo que es un trabajo fantástico y muy especialmente a @ojedathies, un placer volver a trabajar a tu lado.

Podéis ver el resto de preguntas que iremos subiendo en breve en la web https://www.casimedicos.com/mir-2-0/

34 Pregunta vinculada a la imagen nº34

Mujer de 78 años, con antecedentes de cardiopatía isquémica, diabetes mellitus, dislipemia y osteoporosis, que acude a urgencias por traumatismo sobre muñeca izquierda. El estudio radiológico inicial se muestra en la imagen A. Tras reducción ortopédica de la fractura e inmovilización con escayola antebraquial, el control radiológico es el que se muestra en la imagen B. ¿Cuál de las siguientes considera que es la actitud más adecuada a seguir tras este control radiológico?

  • 1. Dado que se trata de una fractura extraarticular, se considera una fractura estable. Se mantendrá la inmovilización durante 2 semanas y a partir de entonces se iniciará programa de ejercicios activos de flexo-extensión y prono-supinación.
  • 2. Dado que la reducción lograda es óptima, se recomienda control radiológico a los 10-15 dias y en caso de no observarse desplazamiento secundario de la fractura, se mantendrá la inmovilización un total de 6 semanas. CORRECTO POR NUESTRAS PARTE Y POR EL MINISTERIO.
  • 3. La fractura de estiloides cubital determina la existencia de una inestabilidad radiocubital distal, que exige su reparación quirúrgica en un segundo tiempo. En el mismo acto quirúrgico se debe realizar la osteosíntesis del radio con placa atornillada.
  • 4. Dada la osteoporosis de la paciente, la probabilidad de desplazamiento de la fractura es muy elevada. El  tratamiento más recomendable es quirúrgico de entrada, con colocación de fijador externo asociado a fijación interna con placa atomillada.
Imagen nº34 MIR 2019

COMENTARIO:

En tratamiento ortopédico incruento de una fractura de radio distal que ha precisado reducción es el control en consultas externas y mantener la inmovilización durante 6 semanas (1 INCORRECTO Y 2 CORRECTO).

La lesión de la estiloides cubital está descrito como uno de los factores que nos permiten clasificar una fractura de radio distal como inestable lo que iría a favor de indicar un tratamiento quirúrgico. Sin embargo la indicación quirúrgica se asienta en características tanto de la propia fractura como del paciente para de esta forma podamos decidir la mejor propuesta de tratamiento para el paciente. Caso de reunirse criterios de tratamiento quirúrgico se realiza la osteosíntesis con placa del radio distal y si el fragmento cubital fuera muy grande puede llegar a plantearse su fijación durante el mismo acto quirúrgico (RESPUESTA 3 FALSA).

La fractura que nos presentan no tiene tanta conminución ni déficit de stock óseo (tampoco nos dicen que sea abierta) como para plantear la utilización de un fijador externo (RESPUESTA 4 FALSA).

198 Hombre de 45 años que acude a consulta por dolor en el hombro izquierdo de 6 meses de evolución que ha aumentado de intensidad en las dos últimas semanas. No relaciona el dolor con traumatismo previo. El dolor es de predominio nocturno y empeora con el movimiento. En la exploración presenta un rango de movilidad pasiva completa con disminución de los últimos grados de rotación interna y elevación activa. No se objetiva deformidad, crepitación ni aumento de temperatura. El diagnóstico más probable es:

  • l. Inestabilidad de hombro asociada o laxitud.
  • 2. Rotura del manguito de los rotadores del hombro. NUESTRA RESPUESTA.
  • 3. Capsulitis adhesiva de hombro.
  • 4. Tendinitis calcificante del tendón supraespinoso del hombro. RESPUESTA DEL MINISTERIO. 

COMENTARIO:

–> COMENTARIO ACTUALIZADO TRAS LAS RESPUESTAS DEL MINISTERIO: En esta pregunta tuvimos duda entre la 2 y la 4. Personalmente creo que discernir entre ambas con los datos que se han proporcionado es realmente difícil y entre especialistas tuvimos diferencias. ABAJO SON LAS JUSTIFICACIONES QUE DIMOS ANTES DE VER LA CORRECCIÓN DEL MINISTERIO.

1 FALSA. No nos han dado datos como para que pensemos en inestabilidad (ni episodio previo de luxación ni hablan de datos que nos hagan pensar sobre la posibilidad de laxitud.

3 FALSA. Nos dicen que no hay déficit de la movilidad pasiva.

 RESPUESTA 2 CORRECTA:

El paciente lleva 6 meses con dolor – que ahora se ha acentuado-, sin traumatismo, que empeora con el movimiento pero es de predominio nocturno. Presenta rango de movilidad pasiva completa con limitación de la movilidad activa. Todo ello sugiere una rotura del manguito rotador, y habla contra la tendinitis calcificante (por la cronología), contra la capsulitis adhesiva (por la movilidad pasiva completa) y contra la inestabilidad glenohumeral (el paciente no refiere ningún síntoma de inestabilidad como luxaciones, aprensión etc, ni hay nada que sugiera laxitud articular). Citando el tratado «Traumatología y Ortopedia para el Grado en Medicina», dirigido por el Prof. Fernando Marco. Ed. Elsevier, ISBN 978-84-8086-677-4 (2015).

 
Rotura del manguito rotador: «El dolor se exacerba con el uso del brazo, especialmente por encima del hombro, y es más agudo de noche y frecuentemente interrumpe el sueño… El recorrido articular pasivo suele estar conservado, mientras que el activo se limita bien por debilidad real o por dolor.» (pág 212).


Hombro congelado: «Los pacientes refieren una pérdida de función gradual del hombro con molestias que les dificultan dormir sobre ese lado. La limitación de movilidad activa y pasiva es el signo clave con variabilidad respecto a su intensidad. Suele percibirse un tope firme con dolor en el extremo del arco de movilidad remanente. Las rotaciones externa e interna y la abducción son las más afectadas. El dolor es continuo, de predominio nocturno, puede irradiarse y llega a alcanzar en ocasiones una gran intensidad…» (pág 214). 

Tendinitis calcificante: «La etapa aguda durará hasta 2 semanas; la fase subaguda, de 3 a 8 semanas, y la crónica, 3 meses o más… Durante la fase aguda, el dolor es tan intenso que el paciente rehúsa mover su hombro y lo mantiene pegado al cuerpo en rotación interna…En general, el arco de movimiento es limitado por el dolor, los pacientes no pueden dormir sobre el hombro afecto y el dolor se exacerba por la noche. Se describe un arco doloroso situado entre los 70 y 110° de elevación con frecuente sensación de atrapamiento y «enganche»…» (pág 216)

199. ¿Cuál de los siguientes tratamientos le parece más apropiado en la actualidad en una osteonecrosís idiopática de cadera en un paciente de 40 años de edad con dolor moderado y sin colapso de la cabeza femoral?

  • l.  Observación,
  • 2. Descompresión central de la cadera con o sin terapia celular complementaria. NUESTRA RESPUESTA Y DEL MINISTERIO.
  •    3.    Osteotomía de rotación del extremo proximal del fémur.
  • 4. Prótesis total de cadera.

COMENTARIO:

El dolor moderado sin colapso de la cadera femoral sugiere un estadio II, que en cuyo caso estaría indicada una descompresión central de la cadera con o sin terapia celular complementaria (RESPUESTA 2 CORRECTA). La observación está indicada en los estadios 0 – I. Como referencia, recomendamos una tabla en la página 301 del tratado «Traumatología y Ortopedia para el Grado en Medicina», dirigido por el Prof. Fernando Marco. Ed. Elsevier, ISBN 978-84-8086-677-4 (2015). 

200. Un hombre de 44 años de edad, trabajador de la construcción, consulta por un dolor lumbar de dos semanas de duración. Dos años antes consultó por un proceso similar. Se trata de un dolor no irradiado, que mejora con el reposo nocturno. No se acompaña de déficit motor o alteraciones sensitivas en la exploración física, La maniobra de Lasegue es negativa. ¿ Qué exploración complementaria considera indicada?

  • 1 . No hay indicación de realizar ninguna exploración complementaria. NUESTRA RESPUESTA Y DEL MINISTERIO
  • 2. Una radiografía anteroposterior y lateral de columna lumbar.
  • 3. Debe realizarse un HLA B27 a fin de descartar una espondiloartropatía,
  • 4. Una resonancia magnética que nos descartará la presencia de una hernia discal.

COMENTARIO:

No tenemos signos de alarma, ni historia de traumatismo, mejora con el reposo, duración escasa (2 semanas) con episodios similares en el pasado en un trabajador de esfuerzo por lo que estaría indicado el tratamiento conservador sin más. RESPUESTA 1 INCORRECTA.

Bibliografía recomendada:

capítulo 130 («Lumbalgias») del Manual de Protocolos y Actuación en Urgencias del Complejo Hospitalario de Toledo. ISBN 978-84-96835-57-3 (2014):  

«Radiología simple: en pacientes menores de 50 años de edad con un episodio de lumbalgia aguda (< 6 semanas) en ausencia de signos de alarma carece de utilidad para el diagnóstico y el tratamiento de la misma»

201. Un joven de 16 años jugador de baloncesto acude por dolor y bloqueos de rodilla. Las pruebas complementarias nos aportan un diagnóstico de osteocondritis dísecante de rodilla en el cóndilo femoral interno. ¿Cuál de los siguientes tratamientos es INCORRECTO?

  • 1. Realización de microfracturas o perforaciones por artroscopia de lo zona lesionada.
  • 2. Injerto ostcocondral autólogo (mosaicoplastia],
  • 3. Prótesis unicompartimentai de rodilla. NUESTRA RESPUESTA Y DEL MINISTERIO.
  • 4. Transplante de condrocitos autólogos,

COMENTARIO:

A la hora de abordar esta lesión las técnicas quirúrgicas a plantear son diversas por abordaje artroscópico.

Solo en alguien de edad avanzada y con cambios degenerativos nos plantearíamos técnicas protésicas (LA RESPUESTA ES LA 3)

202. Un hombre de 55 años con antecedente de metástasis óseas de origen renal acude a urgencias por dolor intenso en cadera derecha. La radiografía simple muestra una lesión lítica de más de dos tercios del diámetro del fémur en reglón peritrocantérica. Teniendo en cuenta que su esperanza de vida es mayor de 6 semanas, con una.calidad aceptable, ¿qué tratamiento recomendaría?



1. Radioterapia precoz,

2. Descarga completa de la extremidad afecta, caminando con muletas.


3. Fijación preventiva con clavo endomedular,


4. Quimioterapia y radioterapia urgente.



Respuesta correcta: la 3. Fijación preventiva con clavo endomedular

Se emplea la escala de Mirels para predecir el riesgo de fractura patológica. Se indica fijación interna si la puntuación es mayor o igual a 8. 

– Lesión lítica – 3 puntos (máximo)

– > 2/3 de diámetro – 3 puntos (máximo)

– Peritrocantérica – 3 puntos (máximo)

– Queda el factor dolor, que puede considerarse de 2 (moderado) o 3 puntos (funcional), en cualquier caso ya supera el umbral. 

Puedes comprobar las respuestas en las siguientes webs:

https://www.casimedicos.com/publicada-la-plantilla-provisional-de-respuestas-del-examen-mir-2019/

https://www.casimedicos.com/respuestas-traumatologia-y-ortopedia-del-examen-mir-2019/

https://www.diariomedico.com/salud/examen-mir-2019-todas-las-preguntas-y-respuestas.html

Preguntas MIR 2015: Cirugía Ortopédica y Traumatología

Un año más he participado en la iniciativa proyecto MIR 2.0 para dar respuestas a las preguntas formuladas en el examen para acceso al programa de formación MIR.

Estas han sido las preguntas que se hicieron ayer en el examen de 2015 de cirugía ortopédica y traumatología. Estos son los enlaces a la web donde están subidas todas las respuestas:

https://wikisanidad.wikispaces.com/Traumatologia+2015

https://wikisanidad.wikispaces.com/MIR+2015

Pregunta 17

trauma9.PNG
Imagen nº 9

Muchacho de 11 años que acude porque desde hace 10 días presenta febrícula diaria de predominio vespertino. En los últimos 2 días la fiebre ha ido aumentando (hasta 38,5 ºC). Tiene poco apetito y ha perdido peso (2 kg). Hace 3 semanas le dieron una patada en la rodilla derecha. Aunque inicialmente sólo tenía dolor posteriormente notó inflamación, que fue en aumento hasta impedirle caminar. Analítica: elevación de PCR (75 mg/L) sin otras alteraciones significativas. Se realiza radiología simple que se muestra.

¿Cuál de los siguientes es el diagnóstico más probable?

  1. Osteomielitis.
  2. Miositis osificante.
  3. Granuloma eosinófilo.
  4. Osteosarcoma.
  5. Osteoblastoma.

Respuesta correcta: 4

Comentario:

Antonio: La imagen es muy mala, he conseguido que un opositor me deje el suyo y lo único que veo es en extremo distal de fémur una masa, yo diría que es un osteosarcoma y marcaría la 4.

Cristina: La correcta es la 4, osteosarcoma. Se ve una masa de partes blandas en torno al fémur distal, con rarefacción de la trabeculación a ese nivel (eso se ve peor, pero es lo que nos da la imagen); se aprecia un triángulo de Codman en cara posterior y lateral del fémur. Edad, sexo, febrícula vespertina, pérdida de peso, aumento de PCR (este último señal de mal pronóstico), masa dolorosa, localización. Lo del golpe y tal es para diagnosticar. Osteomielitis normalmente no da este tiempo de evolución ni localización, aunque siempre cultivamos nuestras biopsias y biopsiamos nuestros cultivos en los tumores óseos infantiles.


Pregunta 18

Pregunta vinculada a la imagen nº9 (disponible en la pregunta 17)
¿Cuál sería el siguiente paso en el manejo clínico de este paciente?

  1. Tratamiento intravenoso y control radiológico en un mes.
  2. Realizar una punción aspiración con aguja fina de la lesión para estudio microbiológico.
  3. Resonancia magnética del fémur y después, si procede, biopsia de la lesión.
  4. Tratamiento antiinflamatorio y control evolutivo clinicorradiológico.
  5. Realizar una gammagrafía ósea con Galio 67.

Respuesta correcta: 3
Comentario:
Antonio: Siguiendo con la idea de que es un osteosarcoma señalaría la 3 para estudiar la lesión.
Cristina: De acuerdo con Antonio, la 3. Hay que hacer RMN del fémur COMPLETO para caracterizar los compartimentos afectos y descartar la presencia de metástasis en el mismo fémur. Aunque sospecháramos osteomielitis, también haríamos probablemente una RMN antes de iniciar antibioterapia. La gammagrafía con galio 67 se usaría para patología infecciosa (pie diabético, infección prótesis), pero en este caso una RMN nos daría mayor rendimiento, también si sería infeccioso (pero es tumoral!)


Pregunta 116

¿Cuál de las siguientes lesiones traumáticas precisa, para evitar complicaciones locales, un tratamiento más precoz?

  1. Luxación traumática posterior de la cadera.
  2. Fractura desplazada del cuello femoral del anciano.
  3. Fractura trocantérica del anciano.
  4. Fractura de cotilo.
  5. Fractura subtrocantérea.

Respuesta correcta: 1
Comentario:
Antonio: Cualquier luxación requiere tratamiento urgente requiriendo ser reducida o correrán riesgo tanto partes blandas como el propio hueso de sufrir necrosis.
Cristina: Luxación traumática posterior de cadera, por la vascualrización precaria y riesgo de necrosis avascular de cadera. La fractura desplazada del cuello femoral del anciano la trataríamos con una prótesis, en cuyo caso no nos preocupa la necrosis avascular de la cabeza femoral. Y en las demás no se pone (tanto) en riesgo la vascularización de la cabeza femoral.


Pregunta 117

La complicación más frecuente de las fracturas del cuello del astrágalo es:

  1. Algodistrofia refleja.
  2. Consolidación viciosa.
  3. Pseudoartrosis.
  4. Osteonecrosis.
  5. Lesión neurológica del tibial posterior.

Respuesta correcta: 4
Comentario:
Antonio: La complicación más frecuente en las fracturas del cuello de astrágalo es la osteonecrosis.
Cristina: Osteonecrosis. La vascularización del astrágalo queda interrumpida en las fracturas desplazadas y su reducción es una urgencia.


Pregunta 119

Chico de 23 años de edad, que al realizar un salto jugando al baloncesto, cae sobre su extremidad inferior derecha con la rodilla en hiperextensión aplicando un giro brusco a su rodilla mientras que mantiene el pie fijo en el suelo. El paciente, percibe un chasquido y dolor agudo en su rodilla, no pudiendo continuar jugando. Nota sensación de inestabilidad al realizar el apoyo de dicha extremidad. En la exploración clínica se aprecia derrame articular intenso por lo que se practica artrocentesis que muestra importante hemartrosis aguda sin presencia de gotitas de grasa sobrenadanado en el líquido extraído. La movilidad de la rodilla está libre y la maniobra de Lachman resulta positiva. ¿Cuál es la sospecha diagnóstica?

  1. Rotura en asa de cubo del menisco interno.
  2. Rotura aislada del ligamento colateral lateral.
  3. Fractura por arrancamiento de la espina tibial anterior.
  4. Fractura unicondílea del cóndilo medial.
  5. Rotura del ligamento cruzado anterior.

Respuesta correcta: 5
Comentario:
Antonio: Rotura del ligamento cruzado anterior. Giro con pie fijo en el suelo, derrame articular intenso y rápido con hemartros y SIN gotas de grasa sobrenadante esto con Lachman positivo es diagnóstico de rotura de LCA.
Cristina: Rotura del ligamento cruzado anterior. Totalmente de acuerdo con la respuesta de Antonio.


Pregunta 201

Después de un fuerte golpe en la rodilla y sobre todo si la extremidad inferior afectada está apoyando sobre el suelo, puede llegar a producirse la llamada «triada desgraciada» que afecta a tres elementos de los componentes anatómicos de la articulación de la rodilla. ¿Cuáles son estos?

  1. Ligamento colateral peroneo, ligamento cruzado posterior y menisco lateral.
  2. Ligamento colateral tibial, ligamento cruzado anterior y menisco medial.
  3. Ligamento colateral tibial, ligamento cruzado posterior y cruzado anterior.
  4. Ligamento cruzado anterior, ligamento cruzado posterior y menisco medial.
  5. Ligamento colateral peroneo, ligamento colateral tibial y cruzado anterior.

Respuesta correcta: 2
Comentario:
Antonio: Ligamento colateral tibial, ligamento cruzado anterior y menisco medial.
Cristina: Esta pregunta es un poco putada porque se refiere a ligamento colateral tibial (=medial) y peroneo (=lateral), según donde se inserta a nivel distal. En la práctica, casi todos hablamos de medial y lateral.

Ha sido un placer colaborar en este proyecto. Espero que a los opositores les ayude nuestro trabajo y a todo aquel que le interesen estos temas.